You are on page 1of 109

UNIVERSITY OF THE EAST

Ramon Magsaysay Memorial Medical Center, Inc.


(UERMMMCI)

COLLEGE OF MEDICINE
DEPARTMENT OF CLINICAL NEUROSCIENCES

NEUROLOGY I and II
Preceptorials

Course Syllabus

1|P a ge
DEPARTMENT OF CLINICAL NEUROSCIENCES
FACULTY

Erman C. Fandialan, MD, FPCP, FPNA


Chairman, Department of Clinical Neurosciences

Section of Neurology

Erman C. Fandialan, MD, FPCP, FPNA Marietta C. Olaivar, MD, FPCP, FPNA
Head, Section of Neurology Residency Training Officer, Section of Neurology

Pia Teresa A. Camara-Chua, MD, DPCP, FPNA Rio Carla F. Pineda, MD, FPNA
Ma, Alma E. Carandang-Concepcion, MD, FPNA Rene B. Punsalan, MD, FPNA
Lex Lycurgus M. Castillo, MD, FPNA Ma Luisa Gwenn F. Pabellano-Tiongson, MD, FPNA
Evelyn Chua-Ley, MD, FPNA Maria Felicidad A. Soto, MD, FPNA
Belinda Lioba L. Mesina-Nepomuceno, MD, FPNA Ma. Katrina Margarita A. Zialcita, MD, FPNA

Section of Neurosurgery

Alfredo L. Tan, MD, FAFN Asis S. Encarnacion, Jr. MD, FAFN


Head, Section of Neurosurgery Residency Training Officer, Section of Neurosurgery

Allan Renan Acosta, MD, FAFN Levi M. Maliwat, MD, FAFN


Christopher O. Concepcion, MD, FAFN Elmer Jose A. Meceda, MD, FAFN
Kathleen P. King, MD, DPBNS Ludovico Villaruel, MD, FAFN

2|P a ge
University of the East
RAMON MAGSAYSAY MEMORIAL MEDICAL CENTER, Inc.
Aurora Boulevard, Quezon City

NEUROLOGY I Preceptorials

LEARNING OUTCOMES

At the end of the course, the student must be able to:

1. Learn the different parts of the neurological examination.

2. Perform a complete neurological examination.

3. Perform specific additional neurologic maneuvers/techniques as


appropriate.

4. Apply the findings in the neurological examination to the process of


localization

3|P a ge
University of the East
RAMON MAGSAYSAY MEMORIAL MEDICAL CENTER, Inc.
Aurora Boulevard, Quezon City

NEUROLOGY II Preceptorials

LEARNING OUTCOMES

At the end of the course, the student must be able to:

1. Develop good attitude in the clinics.


a. Observe punctuality.
b. Exhibit proper decorum and respect for peers and preceptors.

2. Perform correct bedside habits in handling neurologic patients.


a. Develop rapport with the informant.
b. Develop rapport with the patient.
c. Practice communication skills.

3. Get a complete and detailed neurologic history.

4. Perform a complete physical examination and neurologic examination.

5. Localize the lesion.

6. Arrive at an etiologic diagnosis and differential diagnoses.


a. Explain the basis for assessment.
b. Explain the basis for ruling out the differential diagnoses.

7. Discuss the patient’s manifestations and disease process in relation to the basic disciplines:
a. Anatomy
b. Pathophysiology
c. Microbiology
d. Pharmacology

8. Apply the problem oriented approach in formulating management plans in terms of:
a. Diagnostics
b. Support plans
c. Treatment

9. Organize the data gathered into a legible, grammatically correct, accurate and concise case
discussion.

10. Review the case in a preceptorial session.

11. Participate actively in the case discussion.

4|P a ge
University of the East
RAMON MAGSAYSAY MEMORIAL MEDICAL CENTER, Inc.
Aurora Boulevard, Quezon City

NEUROLOGY I and II Preceptorials

REMINDERS TO STUDENTS

1. The students’ evaluation sheet will be kept by the preceptor. If questions arise regarding their
grades, the preceptors will show the evaluation sheet for reference.

2. Allowable absence for any student is 20% of the total required class hours. An absent student
must present an excuse letter from the Infirmary. More than 20% of absence means an
incomplete grade. The student will have to make arrangements with the preceptor for
completion.

3. Rules on attendance shall be governed by the rules in the Students’ Handbook.


a. A student shall be considered absent if he is not present within the:
i. First 20 minutes of a 60-minute class.
ii. First 30 minutes of a 90-minute class.
iii. First 40 minutes of a 120-minute class.
b. A student is late or tardy and receives a half absence if he arrives in class during the first
third fraction of the scheduled class.

NOTE: The highest grade that can be given to a student is 94 and the lowest is 75.

4. In cases where the preceptor will be late for class, students should not leave the classroom until
the first third fraction of the scheduled class has passed.

5. The students should utilize their ward work time to interview and examined assigned patients in
the Neurology ward.

5|P a ge
WAIVER

KNOW ALL MEN BY THESE PRESENTS:

That I, _________________________________, Filipino, a resident of


___________________________________________________________________________________, a
___________ (year in medical school) student of UERMMMCI, currently enrolled for the ______ semester
__________ (school year), do by these presents, acknowledge:

1. That I have been informed during the orientation regarding the Department of Clinical
Neurosciences Policy on Grading System, based on the satisfaction of the following academic
requirements, namely, written exams, class participation, submission of papers, OSCE and class
attendance,

2. That I have been likewise informed during the Orientation about the Department’s Policy on
Absences particularly on the CHED ruling on Failure due to Absences (FA), whereby total absences
constituting 20% or more of the total required total attendance will result to FA (Failure due to
Absences),

3. That the College of Medicine’s Policies are likewise written in the Student’s Handbook ad
current/present Policies supersedes previous policies,

4. That it is my responsibility to inquire from the faculty involved or Department Head about my
deficiencies; and

5. That I endeavor to refer to and read the Student’s Handbook regarding the Department’s policies
on excused and unexcused absences as well as the conduct of special examinations.

IN WITNESS WHEREOF, I have set my hand at _____________________ this _____ day of _________
2018

________________________________
Affiant
Signature over Printed Name

SIGNED IN THE PRESENCE OF:

_______________________________ _______________________________
Signature over Printed Name Signature over Printed Name
(WITNESS) (WITNESS)
6|P a ge
University of the East
RAMON MAGSAYSAY MEMORIAL MEDICAL CENTER, Inc.
Aurora Boulevard, Quezon City

HISTORY FORMAT
(Adapted from Medicine PD II Guidelines)

PATIENT PROFILE

• Purpose:
1. To discover what stimuli in the patient’s environment may be contributing to his illness.
2. To determine factors that may significantly influence diagnostic or therapeutic program for
the patient (ex. Financial resources).
3. To discover some information that may give important clue as to the cause of the patient’s
illness.

• The patient profile reveals the individual as a whole -- his personality, his mental make-up and his
reaction to his environment and his illness.

• Information to be included in the patient profile


1. Life History
a. Name h. Education
b. Sex i. Socioeconomic status of the family
c. Age and birth date j. Family composition and his/her place in the family
d. Place of residence k. Living environment – community, neighborhood,
e. Birthplace/ place of origin basic facilities
f. Race l. Hobbies and interests
g. Religion
2. Marital Status
- History, compatibility, adjustment
3. Occupation and Employment History
a. Nature of present and previous work
b. Presence of occupational hazards
c. Adjustment to working situations
4. Financial Status
5. Personality type and reaction to environment
- High Strung, chronic worrier, easy going, etc.
6. Habits and Description of average day
a. Dietary and eating habits d. Use of tobacco (pack years)
b. Sleeping habits e. Alcohol intake
c. Exercise f. Caffeine and drug intake (substance abuse)
7. Current medications, if any
- Dose, frequency and duration of intake

7|P a ge
SOURCE AND RELIABILITY
• Numerical grade or excellent-good-fair-poor

CHIEF COMPLAINT:
• Concise statement of complaint with duration
• Main reason for seeking consult
• Serves as a guide to the more detailed history

Guidelines in recording the Chief Complaint:


1. It is limited to a brief statement.
2. It is restricted to a single symptom or two at most.
3. It uses the patient’s own words as nearly as possible.
4. It refers to a concrete complaint (symptom). Do NOT use vague phrases such as kidney
trouble or heat trouble.
5. It includes the duration of the symptom.
6. It avoids the use of diagnostic terms or diseases.

HISTORY OF PRESENT ILLNESS (HPI)


• Organized, clear, complete, concise, and accurate chronological narration of the development of
the current illness

• 4 Basic Components of HPI:

1. Restatement of the chief complaint with elaboration in greater detail.


2. A history of the present problem from the time of onset.
3. A full description of the current status of the patient.
4. A summary of all significant positive and negative information.

• Note: It is preferable to use a separate paragraph for each chronological period and in that
paragraph analyze all symptoms completely and note positive and negative information
closely related to the symptoms describe. All other significant positives and negatives should
be summarized separately in the last paragraph (4th component of the HPI).
• Day of admission or consultation should be the reference date (period) of the onset and
progression or appearance of other symptoms. It could be several minutes, few hours/several
hours, days weeks, months, or years prior to consultation or admission day.

• SYMPTOM ANALYSIS

o It is important to use a standard method of analyzing a symptom. One basic outline for
analyzing symptoms:
1. Onset
a. Date of onset
b. Manner of onset (gradual or sudden)
c. Precipitating and predisposing factors related to onset

8|P a ge
2. Characteristic
a. Character (quantity, quality, consistency, appearance)
b. Location and radiation (pain, cardiac murmur)
c. Intensity or severity
d. Timing (continuous or intermittent, duration of each, temporal relationship to
other events)
e. Aggravating and relieving factors
f. Associated symptoms
3. Course since onset
a. Incidence
i. Single acute attack
ii. Recurrent acute attack
iii. Daily occurrences
b. Effect of therapy
c. Progress

9|P a ge
____________________________________________________________________________________

The Six Point Check List for HPI


By Benjamin Policarpio, MD

The HPI if elicited thoroughly and accurately will have a predictive diagnostic value of 85% or even more.
How may one be confident that he has adequately accomplished the HPI?

This is done by going through the checklist of six items: 1. Components, 2. Sequence, 3. Temporal
relationships, 4. Analysis of Symptoms, 5. “Time holes”, and 6. ROS, by asking yourself…

1. Do I have all the components? (signs and symptoms)


2. Do I have the correct sequence? (the sequence of components has a telling influence on the diagnosis)
3. Do I have the correct temporal relationships between the problems and S/S? This means not only
sequence but how the signs and symptoms overlap each other within the time frame of present
illness. This is valuable in the finer point of differential diagnosis since a combination of two or more
symptoms of an illness (syndrome) will have many diagnostic possibilities.
4. Have I done an analysis of symptoms of all the components?
5. Are there “time holes” in my HPI; meaning, are there segments of time in the course of illness which
I am not clear about the behavior of a component (gone, better, worse). All components must be
accounted for within the time line of the illness.
6. Have I done a thorough review of systems? This is to cover the other components of the HPI not
elicited in the interview, or other significant but unrelated problems that have to be looked into and
addressed therapeutically.

It is very obvious that all the imaginable problems related to the thoroughness, accuracy, and
dependability of the HPI are covered by these.

_____________________________________________________________________________________

TEMPORAL PROFILE:

The Temporal Profile Diagram


• Follows after the Review of Systems (before the record of the Physical Examination)
• The Temporal Profile of the HPI is a representative diagram of the different components as they
occur within the time frame or course of the present illness and to what extent they overlap each
other during the period

Max
The height of the symptom curve from the baseline will reflect the severity
of the same. The shape, slope of the symptom curve will also depict the behavior
of a symptom or sign over the cause of the illness, from 10 the|onset
P a gtoe consult or
admission.
The legend will be representing 2 or no more than 3 major symptoms of the
illness, with appropriate lines or colors, on the left side, below the diagram.
The “clinical horizon” (CH) and the time lines coincide (are one and the same)
with each other. A symptom curve that rises above it signifies its appearance or
INTENSITY OF
SYMPTOMS

0
Onset Admission
Legend:
_________ Symptom A
TIME FRAME
__ . __ . __ Symptom B (hours, days, weeks, months, as the case may be)
…………….... Symptom C

Guidelines:
1. As much as possible it is best to have the minimum number of symptom line (one or two or at
most three) to represent the temporal profile of several symptoms.
A temporal profile diagram of the HPI which is cluttered defeats its very purpose ie to show at a
glance the relationship of all the components during the course of the illness.
2. Should several symptoms have similar temporal profile, use only one symptom line to represent
all of them.
3. The graphic symptom line can be color coded or represented by symbols such as , ,-
-------- or *****
4. Should there be more than one problem or illness in the HPI, each problem/illness should be
represented by its own separate corresponding schematic diagram/temporal profile.

• The height, shape, slope and sharpness of the symptom curve will serve to depict the symptom’s
severity, acuteness, and tempo over the time frame where it occurs. Some examples:

Gradual onset. Progressively worsening.

Acute onset. Unrelenting course up to admission.

Intermittent (alternately occurring and ceasing)

11 | P a g e
Continuous. Remittent.

Acute episodes. Remittent + periodic

Recurrent (total episode repeats itself at various intervals of time—hours or days.


Eg afternoon fever every 2 or 3 days.

Periodic – Episode occurs in cluster of time in periods of days or weeks or months


with symptom-free period longer than the symptomatic period.

Gradual onset. Continuous. Waxing and waning.

The temporal profile offers a view of the “forest” (course of illness) as well as the “individual trees”
(components). The relationship of all components over the course of the illness is a great and inestimable
value for correct diagnosis.

12 | P a g e
PAST MEDICAL HISTORY
• Convulsions
• Injuries
• Prior Hospitalizations; Prior Surgeries
• Hypertension, Diabetes, Stroke, Thyroid problems, Cardiac problems

FAMILY HISTORY
• Pedigree
• Health status of living relatives
• Cause of death of deceased relatives
• Heredofamilial diseases: cancer, diabetes, hypertension, allergy, mental illness, heart disease

SOCIAL/ENVIRONMENTAL HISTORY
• Educational attainment
• Occupation
• Adequacy of income (as appropriate)
• Typical day of patient (as appropriate)
• Vices: smoking, alcoholic beverage intake, illicit drug use/abuse
• Environment (as appropriate): state of hygiene, access to potable water, electricity, density of
population in neighborhood

EPILEPSY RISK FACTORS (AS APPROPRIATE)


• Birth History -Course of pregnancy, Term, Type of delivery, Condition at birth, Resuscitation,
Congenital anomalies, Neonatal course
• Developmental History
• History of febrile seizures
• History of CNS infections – meningitis, encephalitis
• History of Head Trauma
• Family History of Epilepsy

REVIEW OF SYSTEMS

13 | P a g e
University of the East
RAMON MAGSAYSAY MEMORIAL MEDICAL CENTER, Inc.
Aurora Boulevard, Quezon City

PHYSICAL EXAMINATION FORMAT

GENERAL SURVEY:
• Sensorium, distress, acute or chronically ill

VITAL SIGNS:
• Temperature per axilla/oral/rectal
• Pulse/Heart Rate
• Respiratory Rate
• Blood Pressure

ANTHROPOMETRICS:
• Weight
• BMI

SKIN:
• Color, moisture, turgor
• Rash: distribution, color, pruritus, description
• Lesions: description
• Hemorrhages: petechiae, ecchymoses, hematoma
• Nails, hair

EYES:
• Eyelids: ptosis
• Conjunctivae
• Sclerae
• Cornea
• Pupils
• Movements
• Exophthalmos/Enophthalmos

EARS:
• Lesions of external ear
• Discharge
• Tympanic membranes
• Mastoids
• Hearing

14 | P a g e
NOSE:
• Patency
• Mucosa
• Septum
• Discharge
• Sinus tenderness

MOUTH:
• Lips
• Teeth
• Gums
• Mucosa
• Tongue
• Breath
• Salivary glands

THROAT:
• Tonsils
• Posterior pharynx
• Post-nasal drip

NECK:
• Thyroid gland, Vessels, Trachea, Stiffness, Masses

LYMPH NODES:
• Cervical, occipital, supraclavicular, axillary, inguinal, epitrochlear, others

BREAST (as appropriate):


• Masses
• Discharge, others

BLOOD VESSELS:
• Pulses: location, quality

HEART:
• Inspection: precordium
• Palpation: point of maximum impulse, heaves, thrills
• Auscultation: S1, S2, S3, rate and rhythm
• Adventitious sounds: murmurs, gallop, clicks

CHEST and LUNGS:


• Inspection: shape, symmetry, respiration, others
• Palpation: fremitus
• Percussion
• Auscultation: breath sounds, adventitious sounds

15 | P a g e
ABDOMEN:
• Inspection: contour, scars, peristalsis, blood vessels, hernia
• Auscultation: presence and number of bowel sounds
• Percussion
• Palpation: tenderness, rigidity, fluid, liver and spleen, kidneys
• Bladder

GENITALIA (as appropriate):


• Male: penis, scrotum and testes, discharge, epididymis
• Female: vulva, discharge

MUSCULOSKELETAL:
• Posture
• Deformities of extremities, spine
• Range of motion

16 | P a g e
University of the East
RAMON MAGSAYSAY MEMORIAL MEDICAL CENTER
Aurora Boulevard, Quezon City
DEPARTMENT OF CLINICAL NEUROSCIENCES

THE NEUROLOGICAL EXAMINATION

The neurological examination is the cornerstone in the evaluation of a neurological patient. If it


is well done, the anatomical localization of the lesion can be established without going into the expensive
high technology imaging techniques. The neurological examination can however be very tedious and long
and it has turned off a lot of students and doctors to the detriment of the patient. It is our contention
that the neurological examination can be made simple and yet adequate enough to localize the lesion.

Objectives:

1. To review the different parts of the neurological examination.


2. To review techniques of eliciting the abnormalities.

Purpose of a Complete Neurologic Examination:

1. To facilitate communication
2. To provide a baseline.
3. To direct testing and guide the physician on what laboratory tests to request for the patient.
4. To localize the lesion.

Instruments for a neurological examination:

Must have: Pocket snellen chart


Neurological hammer
Pen light
Disposable pin/toothpicks/cotton
Tongue depressor
Tuning fork (128 or 256 cps)
Stethoscope
BP apparatus
Ophthalmoscope
Otoscope

Parts of a Neurological Examination:

1. Mental status examination


2. Cranial nerve examination
3. Motor examination
4. Sensory examination
5. Examination of Reflexes
6. Cerebellar Examination
7. Meningeal Examination

17 | P a g e
Guidelines in the Performance of a Neurological Examination

1. Perform the neurological examination in an orderly fashion.


2. Finish the examinations for one system before starting another.
3. Always examine the left and the right side of the body and compare.
4. Compare proximal against distal findings.

NEUROLOGIC EXAMINATION (QUICK GUIDE)

Mental Status Examination

I. Orientation

Person
Time of day Day of week Day of month
Month
Kind of place Name of place Floor
Room number

II. Personal Information

Age Birthday Address Telephone Number

III. Fund of Knowledge

President Last President


Vice President Last Vice President
Governor Last Governor
Date of Discovery of the Philippines By whom?
Date of World War I President then?
Date of World War II President then?

Geography
Capital of the Philippines, some provinces
Knowledge of current events

Calculation
5+6= 12 + 9 = 15 + 16 = 79 + 12 =
7–2= 12 – 4 = 30 – 13 = 100 – 17 =
3x2= 7x6= 8x9= 1 x 11 =

Problems
50 + 25 + 10 + 05 + 03 =
6 mangoes cost 54 cents, cost of 9 mangoes
3 packs of cigarettes at 27 cents each – change from $1.00

18 | P a g e
IV. Name of Objects shown

Tie Belt buckle


Button Shoelaces
Key Flash light
Handkerchief Watch crystal
Wrist watch Watch crown or stem
Pen clip

V. Response to Request to

Close eyes
Raise hands
Touch your nose
Put ______ hand on ______ear
Put ______little finger on _____ knee
Put ______little finger on tip of nose and _____ thumb on _____ear

VI. Digit Span (Give each number once, at one digit per second without grouping. Continue
until patient fails digit or order of 2 numbers of some length)

Number forward Number backward


73 59
58 63
269 825
2531 2897
72596 63159
139625 628537
2897531 3294158

Four things to recall: Blue, 75, Glass, Park (repeated until patient knows them)

Recall how many in : 1 min. 5 min. 10 min. 1 hour

Serial 7’s from 100 (write response and underline errors) or

Serial 3’s from 200

Story:
Yesterday St. Joseph’s school burned down. Fortunately no children were in the school
at the time and 3 firemen were overcome with smoke and were brought to the ST. Luke’s
Hospital. (17 items underlined)

VII. Reading
The one best friend a man can have is his dog. The dog is faithful and true. When the day
is done he will curl up at your feet and sleep, content in the knowledge that you are his
friend as well. (25 items underlined)

19 | P a g e
VIII. Write
Name
Spontaneous writing of 8 – 10 word sentence
Dictation: Today is a lovely day in __________
I hope that doctor will make me well soon.
Copy: This is the way I write today.

IX. Drawing
Circle Square Triangle
Clock with all the numbers and the hands at 7:20
Copy interlocking pentagon

X. Gnostic Functions (recognition of objects)


sight______ touch_______ sound______

XI. Practic Functions - “Show how you would”


Comb hair
Brush teeth
Put in gown
Put on lipstick
Light a cigarette

XII. Insight and Judgment

XIII. Proverbs

XIV. Rote = Repetition forward and backward


Counting Alphabet
Days of week

CRANIAL NERVES

I Vanilla Coffee Tobacco


II Visual acuity (with glasses if available but note SC or CC at 35 cm.
Reads print OD____mm high, OS____mm high)
Confrontation fields (4mm white object) Peripheral and central
Fundus; disc, vessels, venous pulse, retina
II, III Pupils: (R) ____mm (L) ___mm
Reaction to light, direct and consensual light reflex
Acommodation
III, IV, VI Primary gaze
EOM Convergence OKN Red Glass
V Light Touch, Pain, Temperature over V1-V3
Motor strength: Masseter and Temporalis
Jaw jerk
V, VII Corneal Reflex
VII Forehead Palpebral fissure Lid closure Nasolabial fold
Movement to corner of mouth showing teeth or laughing

20 | P a g e
VIII Rubbed fingers heard distance from AD____cm AS____cm
Tuning fork head AD:BC ____sec AC ____sec
AS:BC ____sec AC ____sec
With fork at vertex lateralizes to:
IX, X Uvula
Gag
XI SCM
Shrug
XII Tongue
Atrophy
Fasciculation
Deviation
Strength against cheek
Speech La La La Mi Mi Mi Go Go Go
Massachusetts Institute of Technology
Third Riding Artillery Brigade
MOTOR
Strength
0 - no muscle contraction detected
1 - a barely detectable flicker or trace of contraction
2 - active movement of the body part with gravity eliminated
3 - active movement against gravity
4 - active movement against gravity and some resistance
5 - active movement against full resistance without evident
fatigue. This is normal muscle strength

Shoulder: Flexion Hip: Flexion


Extension Extension
Abduction Abduction
Adduction Adduction
Internal Rotation Internal Rotation
External Rotation External Rotation

Elbow: Flexion Knees: Flexion


Extension Extension

Wrist: Flexion Ankle: Dorsiflexion


Extension Plantar flexion
Pronation Inversion
Supination Eversion

Thumb: Radial Abduct Toes Flexion


Palmar Abduct Extension
Opposition

Facility of serial opposition of thumb to fingers


Site Left Right

21 | P a g e
Tone
Spasticity
Rigidity
Gegenhalten

Walk on tiptoe
Walk on heels

REFLEXES
Reflexes are usually graded on a 0-4 +scale (DTR’s)
4+ very brisk, hyperactive; often indicative of disease; often associated with clonus
(rhythmic oscillations between flexion and extension)
3+ brisker than average; possibly but not necessarily indicative of disease
2+ average; normal
1+ somewhat diminished; low normal
0 no response

DTR’s Biceps Other Reflexes: Jaw jerk


Triceps Suck Grasp
Brachioradiallis Snout Plantar (Babinski)
Knee Superficial abdominal Clonus
Ankle Cremasteric

SENSORY
Touch
Pin
Position (record error/trials and note degree of movement)
Finger Finger Wrist Elbow
Distal Phalanx Proximal Phalanx
R = at ______ R = at ____ R = at ____ R = at ______
L = at ______ L = at ____ L = at ____ L = at ______

Toes Toes Knees Ankles


Distal Phalanx Proximal Phalanx
R = at ______ R = at ____ R = at ____ R = at ______
L = at ______ L = at ____ L = at ____ L = at ______

Vibration Sense (distal phalanx) (record in seconds)


Toe Finger Mid-dorsum foot Hand Lat. Malleolus
Right _____ ______ ____ ______
Left _____ ______ ____ ______
Ulnar Styloid Mid-Tibia
Right _____ ______ _____
Left _____ ______ _____

Temperature

Graphesthesia

22 | P a g e
Stereognosis: safety pin, paper clip, button, coin
Right ______ ______ ______ ______
Left ______ ______ ______ ______

Two-point discrimination: transversely on distal palm:


Right ______cm
Left ______cm

Lateralization: double simultaneous stimulation (if primary modalities)

Romberg’s

CEREBELLAR

Coordination:
Finger – to – nose Heel – to – shin
Alternating Pronation-Supination Tibial tapping
Hand-Patting Finger tapping

Nystagmus

Posture feet _____cm. apart

Tremor

Gait Natural
Tandem walk along straight line

MISCELLANEOUS

Carotids : common
Internal
Vertebral arteries
Body temperature
Bruits : CCA ICA Subclavian Vertebral
Right _____ _____ _____ _____
Left _____ _____ _____ _____

Eyes
Skull

Head, Neck, Spine


Neck rigidity

Spine (deformity, tenderness, morbility)


Straight leg raising test

23 | P a g e
THE NEUROLOGIC EXAMINATION
THE MENTAL STATUS EXAMINATION (MSE)

Most of the information needed in the examination of the mental status of the patient can be
gathered by the physician from the time he observes the patient walking into his clinic and throughout
the interview. In fact, sometimes a clinician can surmise that the mental status of the patient is “intact”
if the patient himself narrates his complaint and history to the physician and it is deemed relatively
accurate and plausible by the physician.

The way the examination is conducted and the questions that the physician chooses depends
largely on the educational attainment and employment background of the patient. For example, one
would not expect a patient who had not gone to school to be able to perform straightforward calculations.
However, if for example, this patient sells fruits and vegetables at the local market, his ability to calculate
can be assessed by relating it to a common problem that he encounters daily: “Sir, if you were selling
mangoes at P80/kilo and tomatoes at P40/kilo and I was to buy 3 kilos of mango and 2 kilos of tomato,
how much would I owe you?” In the same way, in the assessment of a patient who says that he is a
business firm’s account executive, one would not ask him to perform simple arithmetic problems such as
“2 + 2 = ?”.

Traditionally, the mental status exam has always been taught as compartmentalized to certain
“higher order cortical functions” (See Table 1). However, in practice, we find that it is easier to subdivide
the mental status examination into different lobes of the cerebral cortex. This aids in localization later
on, and helps with the organization and ease of examination.

Suffice it to say that the subdivision of the mental status examination into different lobar
functions in the cortex is not absolute. Though some areas of the cortex have been mapped out and
specific functions have been assigned to these specialized areas, many of the more complex functions of
the brain (such as arousal, verbal processing, etc) cannot be precisely pinpointed to one particular area,
rather, they are attributed to be a function of a network between several areas in the cortex. Therefore,
the format of the mental status examination we present here is not all encompassing, but it is the bare
minimum that is required for an adequate screening of patients.

OUTLINE OF THE MENTAL STATUS EXAMINATION

FRONTAL: sensorium, speech, ability to follow commands, attention span, insight and judgment

PARIETAL: left/right orientation; finger agnosia; calculation; writing; stereognosis; graphesthesia;


hemineglect; praxis

TEMPORAL: orientation to time/place/person; memory; fund of knowledge

OCCIPITAL: ability to identify familiar faces, colors and objects

THE FRONTAL LOBE


24 | P a g e
1. Sensorium
The frontal lobe is most concerned with executive functions. Hence, the ability of the patient to be able
to be aware of the command, understand it and execute it properly is in essence what is being tested in
frontal lobe exams. Though the patient’s sensorium or state of arousal is actually a function of the entire
cortical network’s integrity, we record the patient’s sensorium under the frontal lobe exams because it
sets the tone for all other parts of the examination. Hence, if one begins the reporting of the mental
status examination with “the patient is comatose, with no eye opening; no verbal or motor responses…”
it is understood why the other portions of the mental status examination were omitted. Therefore, the
description of the sensorium in this case, becomes the mental status examination.

By convention, many physicians like to rely on the Glasgow Coma Scale (GCS) to describe the
mental status of their patient. However, it is wise to remember that the GCS was developed to help in
the categorizing and prognostication of patients who suffered from head trauma. Obviously not all
neurologic patients that you will encounter have head trauma. Hence, the GCS should never be used as
a “replacement” for the actual description of the patient’s sensorium or the MSE. However, it may be
used as a guide to describe the patient’s sensorium, i.e. “The patient is awake, follows complex commands
but has no verbal output….” Actually describing the state of arousal of the patient gives a clearer picture
of the patient’s level of sensorium rather than the arbitrary number given by a GCS score.

The description of a patient’s level of arousal is further complicated by use of ambiguous words
such as drowsy, stuporous or obtunded. Universal definitions for these terms are found in neurology
textbooks, but it still appears very subjective and tends to differ between physicians. We then again,
suggest the use of descriptives of the patient’s best response to stimuli, as the examples below:

Drowsy – “The patient is drowsy, with unsustained eye opening to name calling, can follow simple
commands and has one-word answers to questions”

Obtunded – “The patient has unsustained eye opening to repeated shoulder tapping and name
calling, occasionally follows simple commands but no verbal output”

Stuporous – “The patient has unsustained eye opening on sternal rub, does not follow commands
and mumbles incoherently”

2. Speech
With regard to speech and the frontal lobe, what is being assessed is the fluency of the patient. The
Broca’s area or Broadmann area 44 is located in the inferior frontal lobe, and as such, the ability of the
patient to speak fluently is part of the examination of the frontal lobe (See Figure). Dysarthria, on the
other hand, which refers to the problem of articulation of words and syllables uttered by the patient; as
well as dysphonia, which refers to a nasal quality of the voice; are typically noted under the examination
of cranial nerves IX and X.

3. Ability to follow commands


As mentioned above, the frontal lobe deals largely with the executive functions of the patient.
Consequently, his ability to follow commands whether simple, one-step commands (i.e. “Raise your
eyebrows”) or more complex, multi-step commands (i.e. “Raise your right hand and put it over your left
knee”) is assessed here.

25 | P a g e
4. Attention span
A patient’s ability to focus and concentrate on a task given is also assessed under the frontal lobe.
The most common way to do this is to ask the patient to perform Serial 7’s: ask him to subtract 7
repeatedly, beginning from 100 (i.e. 100 – 7 =?; 93 – 7 = ?; 86 – 7 = ?, and so on). However, here in the
Philippines, we are faced with many patients who have not gone to school. Therefore, one must be
creative in assessing a patient’s attention span. Alternative ways of assessing the attention span are: a)
asking the patient to name the days of the week forward from Sunday and then backward; and b) asking
then to name the months in a year from January and then backward from December.

5. Insight and judgment


Insight refers to the patient’s knowledge and understanding of his illness and his acceptance of it;
while judgment refers to his reaction to certain situations as defined by certain social norms. Most of the
time, insight and judgment are not overtly assessed, as again, these may be inferred through the
observation and interview of the patient during history taking. If the patient himself is the one narrating
his history and is accepting of the fact that he needs treatment, it can be surmised that he has good insight.
In the assessment of judgment, one can give the patient certain scenarios (i.e. “You were riding a taxi cab
on your way to work, and once you got in, you noticed that a previous passenger had left his wallet in the
back seat. What would you do?” However, it must be noted that interpretation of the patient’s responses
may be extremely subjective.

THE PARIETAL LOBE

In the neurological examination, no other portion will involve the concept of brain dominance
more than the examination of parietal lobe functions. This is one of the reasons why it is imperative to
include the handedness of the patient in the identifying data. Though about 98% of the population will
have left-brain dominance regardless of handedness, there is still that 2% chance that one may encounter
a patient with right-brain dominance. Hence, knowledge of the functions of the dominant versus a non-
dominant parietal lobe is important. Again, we reiterate that the examination outline here in this manual
is the bare minimum expected for the assessment of a patient’s mental status. If, however, a lesion is
localized to a particular lobe (i.e. left parietal lobe) the student is encouraged to read the chapter
Neurologic Disorders Caused by Lesions in Specific Parts of the Cerebrum in Adams & Victor’s Principles of
Neurology, 10th ed. so that a more complete and precise examination of the cortical lobar functions may
be conducted.

A. Dominant Parietal Lobe – Gertsmann Syndrome


The most characteristic tetrad of symptoms that indicate a lesion in the dominant inferior parietal
lobule is what comprises the Gertsmann Syndrome and is composed of the following:

1. Right-left disorientation - An inability to distinguish right from left


2. Finger agnosia – inability to recognize and name fingers
3. Acalculia – inability to calculate
4. Agraphia – inability to write

B. Non-Dominant Parietal Lobe


1. Anosognosia, hemineglect and sensory inattention

26 | P a g e
One of the most interesting effects of a lesion in the non-dominant parietal lobe is the inability
of the patient to recognize the left part of his body as his own and a “neglect” or “inattention”
toward anything on the left. These patients, when their left hand is raised by the examiner, claim
that this hand is “not theirs” or is “unfamiliar”. Also, when double simultaneous stimulation is
done, i.e. stroking both arms lightly with the fingers, the patient claims only to be touched on the
right side. However, when tested one at a time, he can discern the touch.

Some ways of assessing hemineglect can be to ask the patient to look at a picture:

A patient with left hemineglect would miss seeing the two children on the left side of the
picture and say that he could only see a woman washing dishes with water spilling over the sink.
Sometimes, the simplest way of assessing neglect is to ask the patient to draw a line to bisect a
horizontal bar drawn by the examiner:

A patient with left hemineglect would place a line (gray dashed line) toward the right end on
the bar drawn by the examiner (black bar), because he cannot perceive the left extension of that
bar.

2. Topographic memory loss

27 | P a g e
Patients can sometimes lose the ability to explain or draw up the location of something
familiar (i.e. floor plan of their home). A way to assess this is to ask them to explain how to get
from their bedroom to the kitchen, for example, but this necessitates the presence of a
companion who can confirm the patient’s directions. Otherwise, a route familiar to both the
examiner and patient may be used, such as: “You live in Mezza apartments. Can you tell me how
you got to the UERM Emergency Room from your home?”

3. Dressing and Constructional Apraxias


Though dressing and constructional apraxias can be seen in lesions of either dominant or non-
dominant parietal lobes, more severe deficits are noted when the non-dominant parietal lobe is
affected. From the root word praxis or “practical functions”, adding the prefix a- makes the
simplest definition of apraxia as the loss of practical function. Dressing apraxia refers to the
inability of the patient to perform simple, mundane tasks in dressing such as buttoning a shirt.
Some examiners also include tasks like: tying a neck/bowtie, brushing hair, putting on lipstick, etc.

For constructional apraxia, the task is typically to copy a figure presented by the examiner:

C. Either Dominant or Non-dominant Parietal Lobe


1. Hemineglect
2. Apraxias
As discussed above

Apraxias can also be tested by showing the patient a common object such as a pen, watch,
or cellphone and asking them to name that object and demonstrate its use.

3. Stereognosis
Stereognosis refers to the ability of the patient to identify familiar objects by feel. The
patient is instructed to close his eyes and the examiner tests one hand at a time by placing familiar
objects on the palm of the hand and asking the patient to name it. Use small objects that can fit
within the patient’s palm, like a button, coin, small key or paper clip. The same procedure is then
done on the other hand. It is important to note that some patients with parietal lobe lesions may
complain of hemisensory loss on the contralateral upper and lower extremities. However,
stereognosis may be still be checked accurately in patients who complain of about < 20% sensory
deficit.

4. Graphesthesia

28 | P a g e
Graphesthesia refers to the ability of the patient to perceive letters and/or numbers
written on his palm. It is tested one hand at a time, with the patient’s eyes closed. The examiner
then draws either letters on numbers on the palm, and asks the patient what is being drawn. A
common mistake made by students is drawing upside down since they are usually sitting across
from the patient. A good reminder to the examiner is to reposition himself so that he is beside
the patient, and the drawing on the palm is right side up for the patient. Like stereognosis,
patients with about < 20% sensory deficit can still be tested for graphesthesia accurately.

THE TEMPORAL LOBE

1. Orientation
The patient’s orientation to three spheres: person, place and time is tested under the
temporal lobe. Usually, the most resilient of the three spheres is the orientation to person, as the
patient will least likely not recognize someone who is familiar to him (like a spouse or child),
whereas the patient commonly loses orientation to time, especially if confined in the hospital for
a long period of time.

2. Memory
Memory is also tested in three facets: immediate recall, recent and remote. For
immediate recall, three unrelated objects are shown to the patient and he is immediately asked
to repeat the names of the three objects and then checked again after a few minutes. A tip for
doing this test is to show the three objects at the beginning of the neurological exam, then ask
the patient if he remembers them at the end of your examination.

Recent memory can be tested by asking the patient questions like: “What did you have
for breakfast?”; “Where did you have dinner last night?” and have a companion confirm his
answers.

Remote memory is tested by asking “Where did you grow up?” “Who was your first grade
teacher?” “What was the name of your first pet?”

The patient’s fund of knowledge, to some degree also tests memory: “Who was the first
president of the Philippines?”

“Who is the mayor of Quezon City?”

THE OCCIPITAL LOBE

The occipital lobe is the visual cortex and is concerned with the processing of visual stimuli. Thus,
it is not actually vision that is tested here, but what is being seen. Commonly, we ask patients to name
everyday familiar objects like buttons, watches, pens, etc. In addition, we ask the patient to name his
family members to test his recognition of familiar faces; as well as basic colors to test color perception.

This concludes the basic examination of a patient’s mental status. Again, we encourage the
student to use this examination outline by lobe in order to aid in localization later on. Appendices are
attached, containing the traditional outline of the mental status exam, and as the student studies this, we
encourage him to associate each part with the particular cortical lobe concerned.

Appendices containing the outline for the Mini Mental Status Examination (MMSE) and Montreal
Cognitive Assessment (MoCA) Scale in Filipino are also included, however, the student is to remember
29 | P a g e
that these scales are used in the assessment of patients with cognitive impairment and are not used to
replace the Mental Status Examination (MSE) as described in this manual.

Mental Status Examination

I. Orientation

Person
Time of day Day of week Day of month
Month
Kind of place Name of place Floor
Room number

II. Personal Information

Age Birthday Address Telephone


Number

III. Fund of Knowledge

President Last President


Vice President Last Vice President
Governor Last Governor
Date of Discovery of the Philippines By whom?
Date of World War I President then?
Date of World War II President then?

Geography
Capital of the Philippines, some provinces
Knowledge of current events

Calculation
5+6= 12 + 9 = 15 + 16 = 79 + 12 =
7–2= 12 – 4 = 30 – 13 = 100 – 17 =
3x2= 7x6= 8x9= 1 x 11 =

Problems
50 + 25 + 10 + 05 + 03 =
6 mangoes cost 54 cents, cost of 9 mangoes
3 packs of cigarettes at 27 cents each – change from $1.00

IV. Name of Objects shown

Tie Belt buckle


Button Shoelaces
Key Flash light
Handkerchief Watch crystal
30 | P a g e
Wrist watch Watch crown or stem
Pen clip

V. Response to Request to

Close eyes
Raise hands
Touch your nose
Put ______ hand on ______ear
Put ______little finger on _____ knee
Put ______little finger on tip of nose and _____ thumb on _____ear

VI. Digit Span (Give each number once, at one digit per second without grouping. Continue
until patient fails digit or order of 2 numbers of some length)

Number forward Number backward


73 59
58 63
269 825
2531 2897
72596 63159
139625 628537
2897531 3294158

Four things to recall: Blue, 75, Glass, Park (repeated until patient knows them)

Recall how many in : 1 min. 5 min. 10 min. 1 hour

Serial 7’s from 100 (write response and underline errors) or

Serial 3’s from 200

Story:
Yesterday St. Joseph’s school burned down. Fortunately no children were in the school
at the time and 3 firemen were overcome with smoke and were brought to the ST. Luke’s
Hospital. (17 items underlined)

VII. Reading
The one best friend a man can have is his dog. The dog is faithful and true. When the day
is done he will curl up at your feet and sleep, content in the knowledge that you are his
friend as well. (25 items underlined)

VIII. Write
Name
Spontaneous writing of 8 – 10 word sentence
Dictation: Today is a lovely day in __________
I hope that doctor will make me well soon.

31 | P a g e
Copy: This is the way I write today.

IX. Drawing
Circle Square Triangle
Clock with all the numbers and the hands at 7:20
Copy interlocking pentagon

X. Gnostic Functions (recognition of objects)


sight______ touch_______ sound______

XI. Practic Functions - “Show how you would”


Comb hair
Brush teeth
Put in gown
Put on lipstick
Light a cigarette

XII. Insight and Judgment

XIII. Proverbs

XIV. Rote = Repetition forward and backward


Counting
Alphabet
Days of week

32 | P a g e
Appendices

Figure 1. The Lobes of the Brain

33 | P a g e
Figure 2. Brodmann Areas

Figure 3. Simple Classification Scheme for Aphasia

34 | P a g e
CRANIAL NERVE EXAMINATION
(Adapted from DeMyer, 6th ed.)

Cranial Nerve I: Olfactory Nerve


a. Materials: Use test substances (e.g. coffee, vanilla, cloves) placed in a small opaque
container (odd number of substances so there’s no chance of getting 50-50% and you
won’t be able to decide whether it’s normal or abnormal).
b. General Guidelines: Do not use alcohol or perfume as these substances may irritate the
terminal nerve endings of cranial nerve V.
c. Procedure:
1. Ask the patient to close his/her eyes.
2. Compress one of the patient’s nostrils. Hold the vial in front of the open nostril and ask
the patient to sniff the test substance. Note if the patient can perceive and identify the
test substance.
3. Compress the untested nostril. This time, do not present any test substance but
ask the patient if he/she can perceive an odor. This is done to ensure
attentiveness and remove suggestibility.
4. Repeat Procedure 2 for the untested nostril.
5. Record findings.

Cranial Nerve II: Optic Nerve (Adapted from the 8-part Eye Examination c/o Ophthalmology Department)
35 | P a g e
a. Materials: Pocket Snellen or Jaegger chart, Ophthalmoscope
b. Procedure – visual acuity, confrontational visual field testing, fundoscopic examination

Visual Acuity
Visual acuity refers to an angular measurement relating testing distance to the minimal
object size resolvable at that distance. The traditional measurement of distance acuity
refers to a visual test in which a target subtends a visual angle of 5 minutes of arc when a
subject is 20 feet away from the target. The most basic types of vision are the distance
and near visual acuity tests. Even though they test two different aspects of fine-detail
central vision, both tests share some conventions, such as the use of corrective lenses
and an established order for testing each eye.

• Testing Distance Visual Acuity


1. Ask the patient to stand or sit at a designated testing distance (20 feet from a well
illuminated wall chart).
2. Occlude the poorer eye or the eye in complaint. Ask the patient to make sure that
the occluder is not touching or pressing against the eye. Observe the patient to
make sure there is no conscious or inadvertent peeking.
3. Ask the patient to say aloud each letter or number, or name the picture object on
the lines of successively smaller optotypes, from left to right until the patient
correctly identifies only half the optotypes on a line.
4. Note the corresponding acuity measurement shown on that line of the chart.
Record the acuity value for each eye separately with correction and without
correction. If the patient misses half or fewer than half the letters on the smallest
readable line, record how many letters were missed. (e.g. 20/40 -2) If acuity is
less than 20/20, recheck with a 2.4 mm pinhole.
5. Repeat steps 1 through 4 for the left eye, with the right eye covered.
6. Record as:
CC (With correction) SC (Without correction)
VA OD VA OD
OS OS

• Testing Pinhole Visual Acuity


1. Position the patient and occlude the eye not being tested, as done for the
distance acuity test.
2. Ask the patient to hold the pinhole in front of the eye that is to be tested.
3. Instruct the patient to look at the distance chart through the single pinhole.
4. Instruct the patient to use small hand or eye movements to align the pinhole to
resolve the sharpest image on the chart.
5. Ask the patient to begin to read the line with the smallest letters that are legible
as determined on the previous vision test without the use of a pinhole.
6. Record the Snellen acuity obtained and precede with the abbreviation.

PH OD
OS

• Testing Near Acuity

36 | P a g e
1. With the patient wearing the habitual corrective lens for near and the near card
evenly illuminated, instruct the patient to hold the test card at the distance
specified on the card (usually 14 inches).
2. Ask the patient to occlude the poorer eye or the eye in complaint.
3. Ask the patient to say each letter or read each word on the line of smallest
characters that are legible on the card .
4. Record the acuity value for each eye separately in the patient's chart.
5. Repeat the procedure with the right eye occluded and the left eye viewing the
test chart
6. Repeat the procedure with both eyes viewing the test card
7. Record the binocular acuity achieved:

J OD _________
OS _________

Visual Field Examination


The visual field is that portion of a subject’s surroundings that is visible at any one time.
The visual field properly includes central fixation, conventionally measured by visual
acuity tests, and extrafoveal (or peripheral) vision. Central fixation, or visual acuity, and
the visual field are tested in different-ways and provide information on different aspects
of visual function. Visual field testing measures sensitivity, the ability to detect light
thresholds at different locations. An abnormal field can indicate a problem in the retina,
optic nerve, or visual pathway.

The visual fields are routinely screened with the confrontation fields test. If macular
disease is suspected to be causing a central field visual field defect, a device called an
Amsler grid is used to test the central area of each eye’s visual field. If a visual defect is
detected by screening, further evaluation is conducted by manual or automated
procedures known as perimetry. For our purposes, we shall focus mainly on the screening
procedures.

Performing the Confrontation Fields Test

1. Seat the patient and make sure the eye not being tested is occluded.
2. Seat yourself facing the patient at a distance of about 50 cm. Close your eye that
is directly opposite the patient’s occluded eye.
3. Ask the patient to fixate on your nose.
4. Finger movement. Test one eye at a time. Have the patient cover the other eye.
Hold up your left index finger at the inferior temporal quadrant, outside your own
peripheral visual field. Wiggle your finger and slowly move it toward the central
field of vision. Instruct the patient to say “Now” or “I can see it” as soon as he/she
sees the wiggling finger. Try to match the perimeter of the Pt’s visual field against
your own. Test all quadrants of each eye separately, each time starting at the limit
of the field. Perform the test on the other eye. Record your results.
5. Finger counting. After estimating the patient’s visual field, you can further test
the patient by asking the patient to count the number of fingers presented in each
of the four quadrants of the visual field. Test one eye at a time and instruct the

37 | P a g e
patient to cover the other eye. Start by holding your finger at the outer edge of
the patient’s visual field in the same manner as the initial testing. Next, randomly
hold up 1, 2 or 5 fingers in each of the four quadrants and instruct the patient to
identify the number of fingers seen. Perform the test on the other eye. Record
your results.
a. Test patients who have marked visual loss by waving your hand in each
quadrant individually and asking if the patient perceives the motion.
b. With patients who can only perceive light, test in each quadrant
individually for the ability to correctly determine the direction of light
projection by pointing a transilluminator or penlight toward the pupil
while keeping the patient’s other eye completely shielded.
2. Simultaneous finger counting. Present fingers simultaneously in opposite
quadrants, asking the patient to state the total number, using the following
combinations: 1 and 1, 1 and 2, and 2 and 2. This test can reveal a more subtle
field defect than finger counting in each quadrant separately.

Fundoscopic Examination

Examination of the eye posterior to the ciliary body and lens is important in assessing
overall ocular health and in diagnosing and monitoring specific optic nerve, retinal,
neurologic and systemic disorders. Ophthalmoscopy is the examination of the posterior
segment of the eye, performed with an instrument called the ophthalmoscope. The bright
lights that are used also mean that ophthalmoscopy should follow visual acuity
measurement.

The direct ophthalmoscope is a handheld instrument that consists of a handle and a head
with a light source, a peephole with a range of built-in dial-up lenses and filters, and a
reflecting device to aim light into the patient’s eye. It has a magnification of 15x and
provides an erect, virtual image of the retina. Its field of view is about 5 degrees and it
does not provide stereopsis.

Performing Direct Ophthalmoscopv


1. Direct ophthalmoscopy is performed with the eye that corresponds to the eye being
examined.
2. Focus the ophthalmoscope by twirling the dial for the Rekoss disc. The optimal
focusing lens depends on the patient’s refractive error, the examiner’s refractive
error and the examination distance. Hold the ophthalmoscope with your right hand
and move your right eye towards the patient’s right eye. When looking through the
ophthalmoscope, keep both eyes open.
3. Ask the patient to stare into the distance and to hold his/her gaze steady as you
continue the examination. Check the patient’s red-orange reflex at 2 feet and
approach the patient’s eye slowly. Place your left hand lightly over the patient;s head
to steady movement and lightly raise the patient’s right eyelid to improve
visualization of the fundus. Dial the ophthalmoscope’s focusing lens to clarify the
fundus image. Optimal viewing occurs 2-3 cm from the patient’s eye.
4. As the patient stares at the distance target, the ophthalmoscope is angled 15 degrees
temporal to fixation so that the patient’s optic disc is at or near the first visible field.

38 | P a g e
5. Find the optic disc by following retinal blood vessels as they converge. The arrows
formed by vascular bifurcations point to the optic disc. Depending on the patient’s
refraction, the entire disc or only a portion of it will be visible in anyone view. Assess
the AV ratio and the cup:disc ratio. Look for venous pulsations.
6. From the optic disc, follow the optic disc outward to examine the superonasal,
inferonasal, inferotemporal and superotemporal areas around the posterior pole.
Note the vascular color, caliber, bifurcations, crossings and the surrounding
background. Take note of hemorrhages and exudates if present.
7. Examine the macular area for any irregularities.
8. Repeat steps 1-8 on the other eye.

Reporting the Fundoscopic Examination


• Check Red-orange reflex – indicate where present or absent
• Check the media – indicate if clear or hazy
• Check the disc borders – indicate if distinct or blurred
• Note the cup-disc ratio. Normal cup-disc ratio = 0.3
• Note the arteriole-venule ratio (AVR). Normal AVR 2:3
• Check for hemorrhages, exudates and papilledema.

Cranial Nerves II and III: Pupillary Light Reflex

Pupillary Examinations
The pupil is the window of the inner eye, through which light passes to reach retinal
photoreceptors. Because of its potential to reveal serious neurologic or other diseases,
examination of the pupil is an important element of a thorough ophthalmic evaluation. Pathologic
disorders can alter the size, shape, and location of the pupil, as well as the way the pupil reacts to
light and near-focus stimulation.

Technique of Pupillary Examination


1. Under dim room illumination, ask the patient to maintain fixation on a distant target to
avoid pupilloconstriction due to the accommodation reflex.
2. Observe the color, size and shape of the pupils.
3. Compare the size of the two pupils and record the pupillary size in millimeters.

Performing the Direct and Consensual Pupillary Light-Reflex


1. Direct Light Reflex. Instruct the patient to look at a distant target. Shine a bright hand-
held light into the right eye by approaching it from the side or from below. Do not stand
in front of the patient or allow the patient to look directly at the light, which would
stimulate the near reflex and preclude accurate light-reflex testing. Test each eye
separately.
2. Record the direct pupillary response to light in the right eye in terms of the briskness of
the response and the pupillary size. Direct constriction of the pupil in the eye stimulated
by light is called the direct light reflex.
3. Indirect Light Reflex. Repeat steps 1 & 2 in the right eye while observing for the consensual
or indirect light reflex by noting the response to the light of the non-illuminated (left)
pupil. The rapidity of the response and change in pupil size should normally be equivalent
39 | P a g e
to that seen in the direct light reaction and is graded on the same numeric scale. Test the
left eye also and record your results.

Performing the Swinging Flashlight Test


1. Under dim room illumination with the patient fixating a distance target, illuminate the
patient’s right eye directly with a bright handheld light, in a manner identical to that used
when testing the light reflex. Note pupillary constriction in both eyes.
2. Move the light beam immediately and swiftly over the bridge of the patient’s nose to the
left eye, noting the pupillary response in that eye. Normally, the pupil will either constrict
slightly or remain at its previous size. If Instead, the pupil dilates when the light illuminates
it (i.e., the direct light reflex is weaker than the consensual reflex) a relative afferent
pupillary defect is present, which usually indicates a disorder of the optic nerve or severe,
retinal pathology.
3. Quickly swing the light back to the right eye to evaluate the response. A normal response
is again a mild constriction or no change in size at all. Net pupillary constriction or dilation
is an abnormal response.
4. Repeat steps 1-3 rhythmically, spending equal intervals illuminating each pupil, until it is
clear whether pupillary responses are normal or whether one pupil consistently dilates.
Record your results.

Testing for the Accommodation Reflex


1. Indirectly shine a light below the eyes.
2. Ask the patient to stare at your finger as you move it towards the center of the patient’s
nose.
3. As your finger moves closer to the patient’s nose, observe for convergence of the
patient’s eyes. Note the symmetry of convergence. Observe also for the presence of
pupillary constriction as the eye converge. The presence of pupillary constriction during
convergence is the accommodation reflex. Record your results.

Cranial Nerves III (Oculomotor), IV (Trochlear), VI (Abducens):

Eye movements can be monocular (one eye only) or binocular (both eyes together). Monocular
eye movements are called ductions and six terms are used to describe them:

1. adduction – movement of the eye nasally


2. abduction – movement of the eye temporally
3. elevation – movement of the eye upward
4. depression – movement of the eye downward
5. intorsion – nasal rotation of the superior vertical corneal meridian
6. extorsion – temporal rotation of the superior vertical corneal meridian

Binocular eye movements are described as versions or vergences. Versions are normal binocular
eye movements in the same direction. (example: to the right, to the left, etc..) One muscle of each
eye is primarily responsible for the movement of that eye into a particular field of gaze. These two
simultaneously acting muscles are called yoke muscles, and their movement is said to be conjugate,
that is, they work at the same time to move the two eyes in the same direction. The six positions of
gaze in which yoke muxcles act together are known as the cardinal positions of gaze. They are right

40 | P a g e
and up, right, right and down, left and up, left, left and down. Vergences on the other hand are normal
disconjugate binocular eye movements in which eyes move in opposite directions.

The two primary types of vergences routinely evaluated are convergence. (the movement of both
eyes nasally), and divergence, (the movement of both eyes temporally). Binocular eye movements are
described as versions or vergences. Versions are normal binocular eye movements in the same
direction. (example: to the right, to the left, etc..) One muscle of each eye is primarily responsible for
the movement of that eye into a particular field of gaze. These two simultaneously acting muscles are
called yoke muscles, and their movement is said to be conjugate, that is, they work at the same time
to move the two eyes in the same direction. The six positions of gaze in which yoke muscles act
together are known as the cardinal positions of gaze. They are right and up, right, right and down, left
and up, left, left and down. Vergences on the other hand are normal disconjugate binocular eye
movements in which eyes move in opposite directions. The two primary types of vergences routinely
evaluated are convergence (the movement of both eyes nasally), and divergence, (the movement of
both eyes temporally).

Assessing the Ocular Movements


1. Sit facing the patient. Hold your finger or a small fixation target at eye level 10 to 14 inches in
front of the patient looking in a primary position (straight ahead).
2. Ask the patient to follow the target as you move it into the six cardinal fields and up and down
along the midline. Elevate the upper lid with a finger on your free hand to observe down gaze.
Refer to the diagram below for the directions of movement
3. Note whether the amplitude of eye movements is normal or abnormal in both eyes. Rate the
amplitude for both fields by considering normal amplitude to be 100% and rate lesser
amplitudes accordingly.
4. Note any nystagmus that may be present and record the nystagmus according to its presence,
direction and amplitude in any field of gaze where applicable. (Record presence or absence of
nystagmus under cerebellar examination).

Figure 1.
(from Demyer’s Techniques of Neurological Examination 6th ed.)

Cranial Nerve V (Trigeminal)


The trigeminal nerve is a mixed nerve, with both sensory and motor components, but
predominantly sensory.

41 | P a g e
Testing for Corneal Reflex (Afferent: Cranial Nerve V; Efferent: Cranial Nerve VII)
1. Test the corneal reflex using a wisp of cotton. Ask the patient to look to one side and a little
bit up. Approaching laterally, touch the cornea at the area of the corneo-scleral junction, and
observe for the eye to blink. Bring the cotton directly in from the side to avoid entering the
field of vision. That would cause a visually mediated flinch, not a corneal blink reflex .
2. Repeat this on the other eye.

Testing for touch in the area of cranial nerve V – V1,V2 and V3


1. Instruct the patient to say “touch” in response to each touch by a wisp of cotton.
2. After the patient closes the eyes, lightly brush each area of the three sensory divisions of CN
V with a wisp of cotton. Touch alternate areas and sides of the face randomly.
3. As a rule, if the history indicates a specific area of sensory loss, start sensory testing in a
normal area, to give the patient the experience of the normal sensation. Then start in the
middle of the abnormal area and work outward

Testing for temperature discrimination (Warm and cold tube test)


Materials: Test tubes with warm and cold water
Procedure:
1. State, “I want to see how you can tell warm from cool. Please close your eyes, and I will place
something on your cheek. Tell me whether it is warm or cool.”
2. Fill one test tube with warm water and one with cold. Avoid extremes of temperature. You
want to test temperature discrimination, not how much heat or cold the patient can
withstand.
3. Because pain and temperature receptors somewhat overlap and share a common pathway in
the spinal tract of CN V and in the trigeminothalamic tract, disease generally affects both
modalities. For screening purposes, testing one tests both. Test temperature discrimination
first. If the patient discriminates temperature normally, you don’t need to test for pain
perception (DeMyer, 2004).

Testing for pain sensation


Materials: Disposable pins/toothpicks
Procedure:
1. Show the patient a straight pin with its sharp and dull end. Ask the patient to respond sharp
or dull when you apply the pin. Then have the patient shut the eyes to prevent visual cues.
2. Alternately touch the patient with the two ends of the pin randomly to monitor the patient’s
attentiveness and reliability. Apply the stimulus with the same pressure. Make about three
successive pricks for each stimulus because not all individual pricks will hit a pain-sensitive
spot. Start with a normal area to establish communication, so that the patient knows what to
expect. Test the face and dorsum of the hands and feet; avoid the horny skin of the palms and
soles. Always discard the pin after use (De Myer, 2004).

Testing for the Motor Function of Cranial Nerve V


Materials: None
Procedure: (Note: Perform the test on yourself first)
42 | P a g e
1. Inspection: Inspect the temples and cheeks for atrophy of the temporalis and masseter
muscles. The temporal muscle fills out the temple. Even when the patient bites, the muscle is
difficult to palpate, but after temporalis muscle atrophy, the temple sinks in.
2. Palpation: To test for masseter atrophy, ask the patient to clench the teeth together strongly
and unclench several times, while you simultaneously palpate the muscles of the two sides as
they mound up and relax under your fingertips.
3. Testing for weakness of jaw closure
a. Ask the patient to clench the teeth strongly.
b. Place the heel of one palm on the tip of the patient’s mandible and the other hand
on the patient’s forehead. Press hard on the tip of the mandible. You must brace the
patient’s head with your opposite hand because jaw closure is a very strong
movement and you do not want to test the strength of the neck muscles and jaw
closure at the same time. The principle is to test the strength of one muscle or one
limited set of muscles at one time.
c. If the patient complains of fatigability when chewing, have the patient chew for a
period before testing.
4. Testing for weakness of the lateral pterygoid muscles
1. Ask the patient to forcefully open the jaw. Note whether its tip aligns with the crevice
between the upper, medial incisor teeth.
2. Then ask the patient to move the jaw from side to side.
3. Ask the patient to hold the jaw forcefully to the side as you try to push it back to the
center with the heel of your palm. Brace the Pt’s head by pressing your other hand
against the opposite cheekbone

Cranial Nerve VII: Facial Nerve


The facial nerve is a mixed nerve, with both sensory and motor components, but predominantly
motor.

Testing for Motor strength


Materials: None
Procedure:
1. Inspect the face and note for any apparent facial asymmetry during speech and emotional
expression. Observe for the presence of involuntary movements.
2. Ask the patient to do the following in succession and observe for the symmetry of
movements or the presence of weakness:
a. Wrinkle your forehead
b. Look up at the ceiling
c. Close your eyes and don’t let me open them
d. Show me your smile/show me your teeth
e. Puff out your cheeks
f. Pucker your lips
g. Pull down hard on the corners of your mouth
3. Listen to labial articulations. Ask the patient to say “lalalalala”.

43 | P a g e
(from DeMyer, 6th edition)

Testing for taste (anterior 2/3 of the tongue) – usually indicated if the patient presents with
peripheral facial palsy
Materials: Test substance (salt solution and sugar solution); cotton pledgets
Procedure:
1. Ask the patient to gargle with tap water to wash away any substances that may interfere
with the test substance. . Tell the Pt, “I want to place something on your tongue for you
to taste. Stick out your tongue and keep it out. When you recognize the taste, hold up
your hand.”
2. Place the test solution with a cotton pledget or applicator stick on the right or left half of
the patient’s tongue. Do not allow the patient to return the tongue to the mouth because
the saliva will diffuse the taste stimulus beyond the area selected for testing. Allow 15 to
20 seconds for the substance to dissolve and for the patient to respond. Take note
whether the patient identifies the taste correctly or not.
3. Ask the patient to repeat steps 1 and 2 on the untested half of the tongue.
4. Record your results.

Cranial Nerve VIII (Vestibulo-cochlear nerve)

Screening test
• Do otoscopy to ensure that the external auditory canals are open and that the eardrums are
normal
• Rubbing of fingers – a normal person should be able to hear the sound of rubbing fingers if placed
near the ear.
• Present a vibrating tuning fork to each ear and ask the Pt to compare the loudness.
• If the history suggests a cerebral lesion, test for auditory inattention to bilateral simultaneous
stimuli and sound localization by finger rustling (

Tuning fork tests


A. Weber’s Test: to check for lateralization of sound
Materials: Tuning fork 128 or 256 Hz

44 | P a g e
Procedure:
i. Explain the procedure to the patient.
ii. Place the vibrating tuning fork in on the middle of the patient’s forehead or the vertex of
the skull
iii. Ask the patient if the sound is louder on one side or is heard midline.
iv. Record your results.

B. Rinne’sTest: comparison of bone conduction and air conduction


• Normal bone conduction: 10 seconds
• Normal air conduction: 20 seconds

Materials: Tuning fork


Procedure:
1. Explain the procedure to the patient.
2. Hold the tuning fork near the base of the handle. Place the vibrating tuning fork on the
patient’s left or right mastoid process.
3. Ask the patient whether he or she can hear the vibrating sound of the tuning fork. Once
the sound is heard, instruct the patient to raise a hand or say “None” as soon as the sound
disappears.
4. After the patient gives the signal, immediately place the tip of the tuning fork near the
external auditory canal and ask the patient if he/she can still hear the sound.
5. Repeat the procedure on the untested ear.
6. Record your results using the following format:
Rinne Test AD AC > or < BC (AC- Air Conduction ; BC – Bone Conduction)
AS AC > or < BC

Se
Co
ns
nd
ori
uc
ne
tiv
ura
e
l
Test He
He
ari
ari
ng
ng
Lo
Los
ss
s
To
AB
To
N
NO
OR
Lateralization on Weber Test RM
M
AL
AL
ear
ea
r

45 | P a g e
AC
>B
C BC
Rinne but >A
dec C
rea
sed

Cranial Nerve IX (Glossopharyngeal) and X (Vagus)


Testing the motor function of cranial nerves IX and X
1. Speech
a. During the history, appraises the patient’s speech. Perfectly normal articulation
requires no formal testing. Otherwise, test the articulation mechanisms individually.
b. Test articulation by the soft tissues, the soft palate, tongue, and lips, with the KLM
test.
a. Kuh, Kuh, Kuh tests the function of the soft palate
b. La, La, La tests the tongue
c. Mi, Mi, Mi tests the lips
c. Most infants or young children with delayed speech or dysarthria require a thorough
audiologic examination.
2. Difficulty swallowing (dysphagia)
a. To test for mild to moderate dysphagia, give the patient a glass containing 150 mL of
tap water. The patient should swallow it at a rate exceeding 10 mL/s
b. Any patient with dysphagia may aspirate food or fluids into the lung, causing
aspiration pneumonitis. Avoid the timed swallowing test if the patient gives a history
of aspiration or obvious difficulty swallowing saliva. Deficits in coughing maybe as
important as the impaired swallowing in determining aspiration risk.
2. Neurologic examination of the palate and larynx
a. As the patient says Ahh, inspect the tonsillar pillars for asymmetry as they arch
upward and medially to form the palate. Look at the arch, the arch above, not the
uvula. Study your own palatal action in the mirror.
3. Testing for gag reflex:
o Material: tongue depressor
▪ Procedure: Touch one tonsillar pillar and then the other with a tongue blade.

Cranial Nerve XI (Spinal Accessory Nerve)


a. Material: None
b. Procedure: Testing the sternocleidomastoid and trapezius muscle
1. Place your right hand on the left cheek of the patient, your left hand on his right
shoulder to brace him, and try to force his head to the midline. Say, “Turn your head
to the left and do not let me push it back.” . Repeat with the patient’s head turned to
the right.
2. Place one hand on the patient’s forehead and push backward. With your other hand,
Press forward on vertebrae prominens (C7) to brace the patient.
3. “Try to touch your ears with the tips of your shoulders. Hold them there and don’t let
me push them down.”

46 | P a g e
4. Place your hands on both of the patient’s shoulders and press down. Observe from
the front and back and watch for scapular winging.

Cranial Nerve XII (Hypoglossal Nerve)


a. Materials: None
b. Procedure:

1. Inspection of the tongue at rest


a. Inspect the tongue for hemiatrophy, the most reliable sign of a XII nerve lesion.
b. Palpation may help resolve questionable hemiatrophy. While wearing a rubber glove,
palpate each half of the tongue between your thumb and index finger.
2. Testing tongue motility and deviation
a. Say to the patient, “Stick your tongue straight out as far as possible and hold it there.”
Check for alignment of the median raphe of the tongue with the crevice between the
medial incisor teeth. Check the alignment of your own tongue in a mirror.
b. Ask the patient to move the tongue alternately to the right and to the left and to try to
touch the tip of the tongue to the tip of the nose and then to the tip of the chin. On
protrusion the tongue tip should extend well beyond the teeth.
3. Tongue strength:
a. Have the patient press the tongue against his/her cheek while you press your finger
against the cheek.
4. Involuntary movements of the tongue
a. Rippling of a normal tongue frequently indicates incomplete relaxation. Ask the patient
to make some tongue movements and then inspect it again after the patient relaxes it.
Rippling of one half of the tongue, if that half is weak and atrophic, suggests fasciculations.
b. Ask the patient to hold the tongue protruded and still for 30 seconds.
c. Infants with mental retardation or cerebral palsy often display an action called tongue
thrusting. Whenever the mother puts food into the infant’s mouth, the tongue thrusts it
back out, impairing nutritional input.
5. Dysarthria
a. Interruption of the UMN pathway to the tongue, the corticolingual pathway, causes
dysarthria. Observe for dysarthria during the patient’s spontaneous speech.

MOTOR EXAMINATION
In doing the motor examination, it is important to note the following points:
1. Strength
• Check both distal and proximal muscles
2. Tone
• Hypertonia: commonly seen in subacute or chronic corticospinal lesions
• Hypotonia: commonly seen in LMN lesion or acute UMN lesions
• Rigidity: commonly seen in basal ganglia disease
3. Bulk
• Wasting and extensive atrophy correlates with LMN abnormality
47 | P a g e
4. Fasciculations, tremors or involuntary movements
5. Tenderness: common finding in metabolic/inflammatory muscle disease
6. Grading
0 No muscle contraction is detected
1 A trace contraction is noted in the muscle by palpating the muscle while the patient attempts
to contract it.
2 The patient is able to actively move the muscle when gravity is eliminated.
3 The patient may move the muscle against gravity but not against resistance from the examiner.
4 The patient may move the muscle group against some resistance from the examiner.
5 The patient moves the muscle group and overcomes the resistance of the examiner. This is
normal muscle strength.

Procedure:

1. Note the position of the body that the patient assumes when sitting on the examination table.

2. Proceed in an orderly manner from the rostro-caudal direction. Start with the face, neck,
shoulders, arm, forearm, hand, chest, abdomen, thigh, leg, foot and toe sequence. Always
compare the right and the left sides of the body.

3. Test the flexor strength of the patient’s neck. To test neck flexion, place one hand on the patient’s
forehead and the other hand on the patient’s chest to provide bracing and counter pressure. Start
with the patient’s head strongly extended. Then, ask the patient to flex his neck and push against
your hand on his forehead.

4. Test the extensor strength of the patient’s neck. To test neck extension, place one hand on the
patient’s occiput and the other hand on the patient’s back to provide bracing and counter
pressure. Start with the patient’s head flexed, with the chin on the neck. Then, ask the patient to
extend his neck and push against your hand

5. Testing for pronator drift: Ask the patient to extend and raise both arms stretched out in front of
them. Ask the patient to keep their arms in place while they close their eyes and count to 10.
Watch for pronator drift.

6. To test for shoulder flexion, ask the patient to extend and raise both arms stretched out in front
of them. Push down on the proximal or distal part of the arms as the patient tries to resist.

7. To test for shoulder extension, ask the patient to extend and raise both arms stretched out in
front of them. Push down on the proximal or distal part of the arms as the patient tries to resist.

8. To test for shoulder abduction, ask the patient to hold the arms straight out to the sides. Push
down on the proximal or distal part of the arms as the patient tries to resist.

9. To test for shoulder adduction, ask the patient to hold the arms straight out to the sides. Push up
on the proximal or distal part of the arms as the patient tries to resist.

10. To test for internal rotation of the shoulder, ask the patient to raise his elbow so that the upper
arm is perpendicular to body, with forearm upright. Keeping the elbow in place, ask the patient
to rotate the shoulder so that the forearm and hand move downward to the ground. As the

48 | P a g e
patient does this, place your hand under the forearm and resist the patient’s downward
movement.

11. To test for external rotation of the shoulder, ask the patient to raise his elbow so that the upper
arm is perpendicular to body, with forearm parallel to the ground. Keeping the elbow in place, ask
the patient to rotate the shoulder so that the forearm and hand move upward perpendicular to
the ground. As the patient does this, place your hand on top of the forearm and resist the patient’s
upward movement.

12. To test elbow flexion, ask the patient to tightly flex the right forearm. The examiner’s left hand
braces the patient’s right shoulder while the right hand grasps the patient’s right wrist. Ask the
patient to resist and maintain elbow flexion as you try to extend the forearm. Do the same for the
patient’s left, using your left hand to grasp the wrist.

13. To test elbow extension, ask the patient to tightly flex the right forearm. The examiner’s left hand
braces the patient’s right shoulder while the right hand is placed under the patient’s right forearm.
Ask the patient to extend the elbow and push his forearm against your hand. Do the same for the
left, using your left hand to push against the patient’s let forearm.

14. To test wrist flexion, ask the patient to make a fist and flex the fist. With your fingers, hold the
patient’s fist and ask the patient to resist your effort to extend his fist.

15. To test wrist extension, ask the patient to hold his hand and extend the wrist. Place your palm
over the knuckles of the hand and ask the patient to resist the effort as you try to flex his wrist or
push his wrist downward.

16. To test finger flexion, test the patient’s grip by having the patient your fingers in their fist tightly
and instruct them not to let go you attempt remove them.

17. To test finger extension, ask the patient to hold out his hands with palms facing down and fingers
hyperextended. Examine the extensor strength of the patient’s fingers by asking him to maintain
finger extension by resisting your efforts to flex the finger. Match the strength of the patient’s
fingers using your corresponding fingers.

18. To test finger abduction and adduction, test the intrinsic hand muscles having the patient abduct
or “fan out” all of his fingers. Test each of the patient’s fingers by using your corresponding fingers
(patient’s index finger to examiner’s index finger, etc.). Ask the patient to push against your finger
as you push against his finger. Perform the maneuver or both sides, this way you can test finger
abduction and adduction

19. Test the strength of the thumb opposition by telling the patient to touch the tip of their thumb to
the tip of their pinky finger and ask the patient to resist your efforts to pry the fingers open..

20. Test radial abduction of the thumb. If the palm in horizontal, ask the patient to move his thumb
outward. If the palm is vertical, ask the patient to move his thumb upward. Then, instruct the
patient to resist your effort to move his thumb towards the palm.

21. To test palmar abduction, ask the patient to hold his palm up horizontal against the floor. Then,
instruct the patient to move the thumb 90 degrees upward such that the thumb is pointing up.
Ask the patient to resist your effort to move his thumb towards the palm.

49 | P a g e
22. To test hip flexion, ask the patient to lift the knee upwards and to try to resist your efforts as your
hand attempts to push the knee back down.

23. Test the extension of the hip by instructing the patient to press down on the examiner’s hand
which is placed underneath the patient’s thigh.

24. Test the adduction of the legs by placing your hands on the medial sides of the patient’s knees
and asking them to bring both legs together against your effort to push the knees apart.

25. Test the abduction of the legs by placing your hands on the lateral sides of the patient’ s knees
and asking the patient to move their knees laterally apart against your effort to push the knees
together.

26. With the hip and knee flexed at 90‫( ﹾ‬patient seated), test internal rotation of the knee by grasping
the ankle and asking the patient to abduct the ankle.

27. With the hip and knee flexed at 90‫( ﹾ‬patient seated), test external rotation of the knee by
grasping the ankle and asking the patient to adduct the ankle.

28. With the knee flexed at 90‫ﹾ‬, test flexion of the knee by grasping the patient’s ankle and asking
the patient to resist your efforts to straighten the leg

29. Test extension of the knee by grasping the patient’s ankle and asking the patient to straighten the
leg and resist your efforts as you try to flex the knee.

30. Test dorsiflexion of the ankle by placing your hand on top of the patient’s foot and asking the
patient to pull the foot up towards their face as hard as possible against your resistance.

31. Test plantar flexion of the ankle by placing your hand beneath of the patient’s foot and asking the
patient to press down (as if ‘’stepping on a gas pedal”) as hard as possible against your resistance.

32. Inversion is inward rotation of the foot. Test for ankle inversion by asking the patient to rotate
the foot inward such that the medial side of the ankle elevates.

33. Eversion is the outward rotation of the foot. Test for ankle eversion by asking the patient to rotate
the foot outward such that the lateral side of the ankle elevates.

34. Test for toe flexion by placing your finger beneath the patient’s big toe and asking the patient to
bend the toe against the resistance of your finger.

35. Test for toe extension by placing your finger on top of the patient’s big toe and asking the patient
to move the toe up (towards his face) against the resistance of your finger.

SENSORY EXAMINATION
The general senses tested in the Standard Neurological Examination (NE) consist of touch, pain,
temperature, position, vibration, and stereognosis. The history determines how far to extend the sensory
examination. For screening purposes, test only the dorsum of the hands and feet, in addition to the face.

50 | P a g e
A patient may manifest with a “spinal level” or a complaint of changes in sensory perception along a
dermatomal level and this warrants a more thorough testing of the different modalities of pain, light
touch and temperature. Refer to the dermatomal map of the human body and test the patient along these
levels to determine the most likely area where the deficit begins.
When testing for the different sensory modalities, test the right and the left side separately and
compare the results. Ask the patient regarding the quality of the sensation and quantify if the sensations
are equal. If unequal, ask the patient to quantify the degree of impairment on the abnormal side.

Examination of Light Touch Sensation


The patient is asked to close her eyes while the hands and feet are lightly touched with a tissue
or cotton wisp.

Examination of Pain Sensation


The patient is asked to close her eyes. The examiner should hold the shaft of the pin or toothpick
lightly between the thumb and the index finger and apply the stimulus with the same pressure. Make
about three successive pricks for each stimulus because not all individual pricks will hit a pain-sensitive
spot. Start with a normal area so that the Pt knows what to expect. The examiner randomly alternates
touching the patient over dorsum of the hands and feet with the sharp and dull ends of a paper clip (or
toothpick) while the patient identifies the stimulus.

Examination of Temperature Sensation


Temperature sensation can be tested with a cool piece of metal such as a tuning fork or a test
tube filled with cold water and a test tube filled with warm water. The examiner randomly alternates
touching the patient over the dorsum of the hands and feet with the cold and warm test substance while
the patient identifies the stimulus.

Examination of Joint Position Sense


Ask the patient to relax. The examiner places his thumb and index finger on both sides of the 4th
digit of the patient’s hand to stabilize the distal interphalangeal joint. Separate the digit being tested so
that you do not touch other digits. With the other thumb and index finger, hold the sides of the patient’s
finger to be tested and move the finger up or down. Ask the patient to identify if the finger is moving up
or down. Do the same test for the other hand and for the feet.

Examination of Vibratory Sense


Vibratory sense is the ability to perceive the presence of vibration when an oscillating tuning fork
is placed over bony prominences. A tuning fork of 128 (or 256) cps, with weighted ends, is most frequently
used. Sensation may be tested on the great toes, the metatarsal heads, the malleoli, the tibia, the anterior
superior iliac spine, sacrum, spinous processes of the vertebrae, sternum, styloid processes radius and
ulna, and the finger joints. The tuning fork is struck, and placed on the bony prominence, usually the
dorsum of the great toe interphalangeal joint initially, and held there until he no longer feels the vibration.
Testing should compare side to side and distal to proximal sensation. If vibration is absent distally, the
stimulus is moved proximally to the metatarsophalangeal joints, then the ankle, then the knee, then the
iliac spine, and so forth. Upper extremity areas frequently tested include the distal joints of the fingers,
the radial and ulnar styloids, the olecranon, and the clavicles.

You can also test vibration sense by placing a vibrating the tuning fork near the nailbed of the
patient’s index finger. Place the finger pad of your index finger underneath the patient’s finger and ask
him to immediately report when the vibration stops or when he is unable to appreciate the vibration.

51 | P a g e
Normally, the examiner and the patient will have almost similar timing of cessation of vibration. Record
your results. Repeat the examination of the opposite index finger and on both big toes.

Examination of 2 point Discrimination

Two-point discrimination can be tested with a special pair of calipers, or a bent paper clip,
alternating randomly between touching the patient with one or both points. The patient should be able
to determine or discriminate the presence of two points at a certain distance, as follows: 2-4 mm on the
fingertips, 4-6mm on the dorsum of the fingertips, 8-12 mm on the palm, 20-30 mm on the dorsum of the
hand.

Examination of Graphesthesia

To test graphesthesia, ask the patient to close their eyes and identify letters or numbers that are
being traced onto their palm or the tip of their finger. Use any blunt tip, such as the cap end of a ballpoint
pen or the tip of the handle of the neuro hammer.

Examination of Stereognosis

To test stereognosis, ask the patient to close their eyes and identify various objects by touch using
one hand at a time.

Examination of Tactile Extinction

Test also for tactile extinction on double simultaneous tactile stimulation. Instruct the patient to
close his eyes. Using a wisp of cotton or the fingers, the examiner lightly touches the patient’s cheeks, one
side at a time and asks which side is being touched. Next, both cheeks are touched. The patient should be
able to perceive both stimuli and answers. The procedure is repeated for the hands and feet. The examiner
may repeat the test several times to ascertain if the patient has tactile inattention to simultaneous
bilateral stimuli (double simultaneous stimulation)

The Romberg Sign


When proprioception is disturbed, the patient may be able to stand with eyes open, but sways or
falls with eyes closed. In performing the Romberg test, the patient is asked to stand erect, feet together,
with hands preferably on his sides, initially with his eyes open. The examiner then observes the patient
for any swaying or imbalance. The examiner then asks the patient to close his eyes, and again observe for
any swaying or imbalance.

52 | P a g e
REFLEXES
REFLEXES GRADING
very brisk, hyperactive; often indicative of disease; often
4+ associated with clonus (rhythmic oscillations between flexion and
extension)
brisker than average; possibly but not necessarily indicative of
3+
disease
2+ average; normal
1+ somewhat diminished; low normal
0 no response

Materials: Neurological Hammer


Procedure:
53 | P a g e
1. Check for symmetry of reflexes between right vs left and upper vs lower extremities
2. Check the following deep tendon reflexes:
a. Biceps: The biceps reflex is elicited by placing your thumb on the biceps tendon and
striking your thumb with the reflex hammer and observing the arm movement

b. Triceps: The triceps reflex is measured by striking the triceps tendon directly with the
hammer while holding the patient’s arm with your other hand.

c. Brachioradialis

d. Patella: With the lower leg hanging freely off the end of the chair, the “knee-jerk”
reflex is tested by striking the patellar tendon directly with the reflex hammer.

54 | P a g e
e. Achilles: The ankle reflex is elicited by holding the relaxed foot with one hand and
striking the Achilles tendon with the hammer and noting plantar flexion.

3. Test for Superficial reflexes


a. Superficial Abdominal
b. Anal
c. Cremasteric
d. Bulbocavernosus

4. Test for Pathologic reflexes


a. Babinski

Babinski Variants

55 | P a g e
Chaddock The skin under and around the lateral malleolus is stroked in a circular fashion. The
stimulus may also be carried forward from the heel to the small toe.

Bing The dorsum of the big toe is pricked with a pin. The big toe withdraws into the pin when
abnormal, as opposed to being flexed away from the stimulus when normal.

Oppenheim apply heavy pressure to the anterior surface of the tibia, stroking down to the ankle

Gordon Compressing the calf muscles, or applying deep pressure to them.

Schaefer Pinching the Achilles tendon enough to cause pain

Trapmorton Percuss tendon of the big toe

Gonda- Pull the fourth toe outward and downward for a brief time and release suddenly
Stransky

b. Hoffman’s sign
• “Babinski of the upper extremity”
• Procedure: The Hoffman response is elicited by holding the patient's middle finger between the
examiner's thumb and index finger. Ask the patient to relax their fingers completely. Once the
patient is relaxed, using your thumbnail press down on the patient's fingernail and move
downward until your nail "clicks" over the end of the patient's nail. The extension of the phalanx
stretches the flexor muscles, causing the fingers and thumb to flex.

c. Myerson’s sign
• Patient is unable to resist blinking when tapped on the glabella, the area above the nose and
between the eyebrows
• early sign of Parkinson’s disease

d. Clonus
• Clonus is a rhythmic oscillation of a body part, elicited by a quick jerk.
• Test clonus if any of the reflexes appeared hyperactive. Hold the relaxed lower leg in your hand,
and sharply dorsiflex the foot and hold it dorsiflexed. Feel for oscillations between flexion and
extension of the foot indicating clonus.
• To elicit wrist clonus, simply jerk quickly up on the patient’s hand.

CEREBELLAR EXAMINATION
Materials: None

56 | P a g e
(adapted from De Myer’s Techniques of Neurological Examination, 6th ed.)

Examination of Nystagmus
1. Ask the Pt look straight ahead and place your index fingers in the temporal fields. Ask the Pt to
look first at one finger and then the other and then ask the patient to look rapidly from one to the
other several times. Observe for the presence of jerky movements instead of smooth pursuit.

Examination of Arm Dystaxia


1. Examination of postural tremor
• Ask the patient to extend the arms straight out in front. Inspect the arms for wavering,
indicating incoordination during this volitionally maintained posture, and for frank, rhythmic
postural tremor
2. Examination of dysmetria
• Ask the patient to touch the tip of his index finger exactly to the tip of his nose and then to
exactly touch the tip of the examiner’s index finger. The examiner then moves his index finger
deliberately to reposition the patient’s target. Do this for several repetitions and instruct the
patient to perform the movements as fast as he is able to.
• Observe if the patient overshoots or undershoots the target (dysmetria).
• Inspect for dystaxia of the movement in progress or frank tremor that increases as the
patient’s finger approaches the examiner’s finger (intention tremor).

57 | P a g e
3. Examination of dysdiadochokinesia
• Instruct the patient to hold out his hands. Ask him to pronate and supinate the left hand over
the right palm as rapidly as possible. Repeat the procedure over the left hand. Test the hands
separately and together. The dystaxic hand overshoots one time, undershoots the next, and is
slower than normal.
• The “thigh-patting test” is another method to test for the presence of dysdiadochokinesia.
Instruct the patient to alternately slap the palm and back of the hand on the thigh, as rapidly
and rhythmically as possible. Make sure there is an audible spund heard with each pat. Test
each hand separately and then together. The examiner then sees and hears the slow rate and
dysrhythmia of the ataxic hand.

Examination for Overshooting and Check Reflex


1. Instruct the patient to stand with eyes closed and arms outstretched. The examiner then tells the
patient, “ I am going to tap your arms. Hold them still and don’t let me move them.” Strike the
back of the patient’s wrist with your fingers, strong enough to displace the arm. Observe if the
patient’s arm is able to return quickly to its original position. The normal subject’s arm returns
quickly. The abnormal subject’s arm oscillates back and forth and overshoots several times.
2. Examination of the normal rebound response
• Have the patient extend and hold out his arms in front. Grasp both wrists and instruct the
patient to try to hold them in place as you pull down on them. After a few seconds of pulling
down, suddenly release your grip. Observe as the normal arm flies upward a small distance,
check and rebound, and quickly resume the resting position. The patient with cerebellar
dysfunction overshoots by flying upward a greater distance and then overshoots the up and
down position several times before assuming the original position of the arm
3. The “arm-pulling test” is also done to demonstrate overshooting. With his right arm extended,
the examiner faces the patient and places his right hand over the patient’s right shoulder. The
examiner then asks the patient to flex his right arm. The patient is instructed to pull his right arm
as hard as he can while the examiner pulls the patient’s forearm with his left hand. After a few
seconds, the examiner releases the patient’s forearm. The normal patient is able to check the
arm’s movement. The cerebellar patient fails to check the movement and may hit his face. This is
the reason the examiner’s right arm is extended and placed over the patient’s right shoulder.
Repeat the procedure for the patient’s left arm.

Examination of Leg Dystaxia


1. The heel-to-shin test is performed with the patient seated or supine. Instruct the patient to place
one heel directly on top of the opposite knee and hold the position for a few seconds. Observe
for positional tremor. The next step is to instruct the patient to slide the heel in a straight line
over the shin. After the heel has slid down the entire length of the shin, ask the patient to lift the
heel and repeat the process of sliding the heel along the shin a total of 5 attempts. Repeat the
test on the other side.
2. For the heel-tapping test, ask the Pt to place one heel over the middle part of the other shin and
to tap the shin with the heel as rapidly as possible on one spot.

Examination of Hypotonia
1. At rest, the hypotonic patient assumes floppy postures and joint positions uncomfortable for a
normal subject—rag-doll or dumped-in-a-heap postures. In a normal person, muscle tone helps
to limit joint excursions. (Demyer, 2011)

58 | P a g e
2. When walking, the hypotonic Pt presents a floppy, sagging, loose-jointed appearance. The arms
fail to swing properly, the knees may bend backward slightly (genu recurvatum), and the head
and trunk bob—a rag-doll gait, as seen in drunkenness. (Demyer 2011)

Examination of Gait and Stance


1. Inspect the Pt for swaying when standing, which involves volitional posture, and for dystaxia of
gait, which involves volitional foot placement
2. Instruct the patient to walk normally and observe for the presence of broad-based gait,
imabalance, arm swing, swaying.
3. Instruct the patient to perform tandem-walking. Instruct the patient to walk a straight line with
one foot directly in front of the other, with the heel touching the toe of the other foot as it is
placed in front (like walking on a tight-rope).
4. Instruct the patient to walk on his heels, then next on his toes.

MENINGEAL EXAMINATION
Materials: None
Procedure:
1. Kernig’s Sign: performed with the patient supine and the hips and knees flexed. Knees are then
extended, and pain upon extension of the knees past 135 degrees indicates a positive Kernig’s
sign.
2. Brudzinski’s Sign: performed with the patient supine, the physician’s hand behind the patient’s
head, and his other hand on the patient’s chest. Flexing the patient’s neck will cause the patient
to flex his knees and hips, indicating a positive Brudzinski’s sign.

59 | P a g e
PRINCIPLES OF LOCALIZATION
By: Dr. Rene Punsalan

LOCALIZATION
• It is a diagnostic exercise of determining from the signs (most often) or symptoms of the patient what
site of the nervous system has been affected by a certain disease process
• It is the determination of where in the nervous system the damage (~lesion) has occurred

A. APPROACH TO DIAGNOSIS
1) Data Gathering: Patient history , General physical examination (PE), Neurologic examination (NE)
2) Anatomic Localization
a) “WHERE is the lesion located?”
b) Can be one specific location, be multifocal, or be part of a diffuse process
3) Etiologic Diagnosis
a) “What is the NATURE of the lesion?”

(Adapted from Adam`s Principles of Neurology, 5th Edition)

B. BENEFITS OF LOCALIZATION
1. Localization directs diagnostic work-up
• The symptoms should direct the physician as to the appropriate diagnostic test to order
2. Limits the differential diagnosis
• In turn, this would also limit the diagnostic work-up
3. Estimates the extent / size of the lesion
4. May suggest the etiologic diagnosis
• Guards against going straight to a specific etiologic diagnosis

C. STEPS IN LOCALIZATION
1. List ALL abnormal neurologic signs
2. Determine ALL possible neuroanatomic correlates per sign
3. Find the “intersection”
D. LEVELS OF LOCALIZATION
60 | P a g e
1. Meninges and cerebrospinal fluid (CSF)
2. The brain
(a) Supratentorial structures (above the tentorium cerebelli)
• Cerebral hemispheres
• Basal nuclei (found within the substance of the cerebral hemispheres)
• Diencephalon (thalamus, hypothalamus, subthalamus, & epithalamus)

(b) Infratentorial structures (below the tentorium cerebelli)


• Brainstem (midbrain, pons, & medulla oblongata)
• Cerebellum
3. Spinal cord: identify level and laterality
4. Peripheral nerve
• Cranial nuclei (~brainstem)
• Anterior horn cell (~spinal cord)
• Root (ventral or dorsal)
• Peripheral nerves and cranial nerves
5. Neuromuscular junction (NMJ)
6. Muscle / effectors

E. CHARACTERISTICS OF LESIONS TO BE OBSERVED


1. Extent:
(a) Diffuse: involves both sides of the neuroaxis
(b) Focal: localized to one area
(c) Multifocal: may stimulate a diffused presentation; involves several areas

2. Laterality: left or right

3. Location:
(a) Intra-axial / Intramedullary: (within the substance of the brain or spinal cord,
respectively)
• e.g. Intraparenchymal hemorrhage or infarct, brain tumor (such as glioma)
• Is it focal: Cerebrum? Ventricular cavities or passage ways? Basal nuclei / ganglia?
Brainstem, cerebellum, or SC?
• Is it multifocal or diffuse?

(b) Extra-axial / Extramedullary: (outside the brain parenchyma or spinal cord,


respectively)
• e.g. Meningioma, meningitis
• Extramedullary intradural: inside canal but outside spinal cord substance
• Extramedullary extradural: outside spinal cord and outside the dura (may be
involving bones)
• In a meningeal or bony covering?
• In a meningeal space: epidural, subdural or subarachnoid?
• In a nerve root, plexus, peripheral nerve, neuromuscular junction, or muscle?

61 | P a g e
University of the East
RAMON MAGSAYSAY MEMORIAL MEDICAL CENTER, Inc.
Aurora Boulevard, Quezon City
NEUROLOGY I and II Preceptorials
School Year 2018-2019

REMINDERS TO STUDENTS

1. The students’ evaluation sheet will be kept by the preceptor. If questions arise regarding their
grades, the preceptors will show the evaluation sheet for reference.

2. Allowable absence for any student is 20% of the total required class hours. An absent student
must present an excuse letter from the Infirmary. More than 20% of absence means an
incomplete grade. The student will have to make arrangements with the preceptor for
completion.

3. Rules on attendance shall be governed by the rules in the Students’ Handbook.


a. A student shall be considered absent if he is not present within the:
i. First 20 minutes of a 60-minute class.
ii. First 30 minutes of a 90-minute class.
iii. First 40 minutes of a 120-minute class.
b. A student is late or tardy and receives a half absence if he arrives in class
during the first third fraction of the scheduled class.

NOTE:
The highest grade that can be given to a student is 94 and the lowest is 75.

4. In cases where the preceptor will be late for class, students should not leave the classroom
until the first third fraction of the scheduled class has passed.

62 | P a g e
University of the East
RAMON MAGSAYSAY MEMORIAL MEDICAL CENTER, Inc.
(UERMMMCI)
Aurora Boulevard, Quezon City

NEUROLOGY I Preceptorials
School Year 2018-2019

STUDENT’S EVALUATION SHEET

Name of Student:_____________________________ Preceptor: _____________________


Inclusive Dates: _____________________________ Final Grade: _____________________

Date
Grading System
1. Attendance (15%)
2. Return Demonstration (65%)
- Complete Pertinent Neurological
Examination
3. Participation in Preceptorials (20%)
TOTAL GRADE

With my knowledge:

____________________________ ________________________________
Student’s Signature/Date Preceptor’s Signature/Date

63 | P a g e
University of the East
RAMON MAGSAYSAY MEMORIAL MEDICAL CENTER, Inc.
(UERMMMCI)
Aurora Boulevard, Quezon City
NEUROLOGY I Preceptorials
School Year 2018-2019

Name of Student: _______________________________________ Preceptor: ______________________


Dates: ________________________________________________ TOTAL Grade: ___________________

GRADING RUBRIC FOR PRECEPTORIAL CLASSES

Criteria 4 3 2 1 Total
Student is punctual Student is present Student is present Student is absent for
and present for but less than 15 but more than 15 preceptorial class
Attendance
preceptorial class minutes late for minutes late for
preceptorial class preceptorial class
Student always has Student usually Student rarely has Student never
something RELEVANT has something something contributes any
to contribute to the RELEVANT to RELEVANT to relevant and
Level Of Engagement
group discussion by contribute to the contribute to the substantial data to the
In Small Group
sharing ideas, asking group discussion group discussion by group discussion by
Discussions
questions, or making by sharing ideas, sharing ideas, asking sharing ideas, asking
plans. asking questions, questions, or questions, or making
or making plans. making plans. plans.
Student listens when Student listens Student does not Student does not
others talk and when others talk. listen when others listen when others
Listening Skills incorporates or talk. talk and often
builds off of the ideas interrupts when
of others. others speak.
Student is very fluent Student shows Average fluency – Student has poor
– speaks in a clear, good command of speaks in a verbal fluency --
concise, well- the English reasonably generally finds
Verbal Communication
organized manner. language -- better organized manner; difficulty in expressing
Skills
than average sometimes fails to his/her thoughts.
ability to get his get points across
ideas across. well.
Student ALWAYS Student MOST OF Student Student NEVER shows
shows respect and THE TIME shows OCCASIONALLY respect and
professionalism in respect and shows respect and professionalism in
Behavior dealing with peers, professionalism in professionalism in dealing with peers,
preceptors and dealing with dealing with peers, preceptors and
patients peers, preceptors preceptors and patients
and patients patients
Student is almost Student is usually Student is rarely Student is never
Preparation for always prepared for prepared for prepared for prepared for
Preceptorial Class preceptorial case preceptorial case preceptorial case preceptorial case
discussions discussions discussions discussions
Total

64 | P a g e
University of the East
RAMON MAGSAYSAY MEMORIAL MEDICAL CENTER, Inc.
(UERMMMCI)
Aurora Boulevard, Quezon City

NEUROLOGY II Preceptorials
School Year 2018-2019

STUDENT’S EVALUATION SHEET

Name of Student:_____________________________ Preceptor: _____________________


Inclusive Dates: _____________________________ Final Grade: _____________________

Date
Grading System
4. Attendance (15%)
5. Case Presentation (65%)
- Complete Neurologic History
- Complete Pertinent PE and NE
- Temporal Profile
- Neurologic Localization
- Case Discussion (Etiologic diagnosis
and differential diagnosis)
6. Participation in Preceptorials (20%)
TOTAL GRADE

With my knowledge:

____________________________ ________________________________
Student’s Signature/Date Preceptor’s Signature/Date

65 | P a g e
University of the East
RAMON MAGSAYSAY MEMORIAL MEDICAL CENTER, Inc.
(UERMMMCI)

Aurora Boulevard, Quezon City


NEUROLOGY II Preceptorials
School Year 2018-2019

Name of Student: _______________________________________ Preceptor: ______________________


Dates: ________________________________________________ TOTAL Grade: ___________________

GRADING RUBRIC FOR WRITTEN REPORTS

Criteria 4 3 2 1
Total
Written patient Written patient Written patient Written patient
history is history is adequate history shows history is incomplete
History Taking
comprehensive and but shows some numerous and inaccurate
accurate inconsistent data inconsistent data
Written clinical Written clinical Written clinical Written clinical
examination is examination is examination is examination is
Clinical
comprehensive and Comprehensive numerous incomplete and
Examination
accurate but shos some inaccurate data inaccurate
inaccurate data
Case analysis is Case analysis Case analysis Case analysis is
adequate, shows shows numerous inadequate and
Analysis of Case comprehensive and inadequacies inadequacies reflects poor critical
(Case Discussion) reflects critical thinking of the
thinking of the student
student
Writes exceedingly Writes very well, Writes reasonably Poor written
well; thoughts clear rarely requiring well, requiring communication skills;
Written and well-organized; rework in syntax or rework both in manifests difficulty in
Communication no grammar and grammar, work grammar and organizing thoughts,
Skills syntax (sentence organized and syntax; some as well as syntax and
construction) lapses clear thoughts are not gramma problems
expressed clearly

Total

66 | P a g e
University of the East
RAMON MAGSAYSAY MEMORIAL MEDICAL CENTER, Inc.
(UERMMMCI)
Aurora Boulevard, Quezon City
NEUROLOGY II Preceptorials
School Year 2018-2019

Name of Student: _______________________________________ Preceptor: ______________________


Dates: ________________________________________________ TOTAL Grade: ___________________

GRADING RUBRIC FOR PRECEPTORIAL CLASSES

Criteria 4 3 2 1 Total
Student is punctual Student is present Student is present Student is absent for
and present for but less than 15 but more than 15 preceptorial class
Attendance
preceptorial class minutes late for minutes late for
preceptorial class preceptorial class
Student always has Student usually Student rarely has Student never
something RELEVANT has something something contributes any
to contribute to the RELEVANT to RELEVANT to relevant and
Level Of Engagement
group discussion by contribute to the contribute to the substantial data to the
In Small Group
sharing ideas, asking group discussion group discussion by group discussion by
Discussions
questions, or making by sharing ideas, sharing ideas, asking sharing ideas, asking
plans. asking questions, questions, or questions, or making
or making plans. making plans. plans.
Student listens when Student listens Student does not Student does not
others talk and when others talk. listen when others listen when others
Listening Skills incorporates or talk. talk and often
builds off of the ideas interrupts when
of others. others speak.
Student is very fluent Student shows Average fluency – Student has poor
– speaks in a clear, good command of speaks in a verbal fluency --
concise, well- the English reasonably generally finds
Verbal Communication
organized manner. language -- better organized manner; difficulty in expressing
Skills
than average sometimes fails to his/her thoughts.
ability to get his get points across
ideas across. well.
Student ALWAYS Student MOST OF Student Student NEVER shows
shows respect and THE TIME shows OCCASIONALLY respect and
professionalism in respect and shows respect and professionalism in
Behavior dealing with peers, professionalism in professionalism in dealing with peers,
preceptors and dealing with dealing with peers, preceptors and
patients peers, preceptors preceptors and patients
and patients patients
Student is almost Student is usually Student is rarely Student is never
Preparation for always prepared for prepared for prepared for prepared for
Preceptorial Class preceptorial case preceptorial case preceptorial case preceptorial case
discussions discussions discussions discussions
Total

67 | P a g e
University of the East
RAMON MAGSAYSAY MEMORIAL MEDICAL CENTER, Inc.
(UERMMMCI)
Aurora Boulevard, Quezon City

NEUROLOGY I OSCE
School Year 2018-2019

Name of Student: _______________________________________ Preceptor: ______________________


Dates: ________________________________________________ TOTAL Grade: ___________________

GRADING RUBRIC FOR OSCE NEUROLOGIC EXAMINATION TASK

PARAMETERS SCORE
TASK 3 2 1 0
Speed, style and Speed, style and Examination done Examination was
technique are technique are is appropriate, but not done OR is
CLINICAL TASK adequate adequate with is clumsy and inappropriate for
(3 points) minor deficiencies confused; slow, what the task
unsure and with required
major deficiencies
TOTAL SCORE

68 | P a g e
University of the East
RAMON MAGSAYSAY MEMORIAL MEDICAL CENTER, Inc.
(UERMMMCI)
Aurora Boulevard, Quezon City

NEUROLOGY II OSCE
School Year 2018-2019

Name of Student: _______________________________________ Preceptor: ______________________


Dates: ________________________________________________ TOTAL Grade: ___________________

GRADING RUBRIC FOR OSCE NEUROLOGIC EXAMINATION TASK

PARAMETERS SCORE
TASK 3 2 1 0
Speed, style and Speed, style and Examination done Examination was
technique are technique are is appropriate, but not done OR is
CLINICAL TASK adequate adequate with is clumsy and inappropriate for
(3 points) minor deficiencies confused; slow, what the task
unsure and with required
major deficiencies
TOTAL SCORE

GRADING RUBRIC FOR APPROACH TO NEUROLOGIC DISEASES TASK

PARAMETERS SCORE
TASK 3 2 1 0
Laterality and Laterality given is Laterality given in No answer given
Localization given Correct but CORRECT but OR Laterality and
are both Localization is Localization given Localization given
NEUROLOGIC
CORRECT INCOMPLETE or in INCORRECT OR are INCORRECT
LOCALIZATION
PARTIALLY Laterality is
(3 points) CORRECT INCORRECT but
Localization is
CORRECT
TASK 2 1 0
Etiologic diagnosis Etiologic diagnosis No Answer is
given is CORRECT given is CORRECT given or Answer is
ETIOLOGIC DIAGNOSIS
and COMPLETE but INCOMPLETE COMPLETELY
(2 points) or PARTIALLY INCORRECT
CORRECT
TOTAL SCORE

69 | P a g e
University of the East
RAMON MAGSAYSAY MEMORIAL MEDICAL CENTER, Inc.
(UERMMMCI)
Aurora Boulevard, Quezon City

CHECKLIST OF PSYCHOMOTOR SKILLS FOR NEUROLOGY I

Name of Student: _____________________________________ Preceptor: __________________

NEUROLOGIC EXAMINATION YES NO


MENTAL STATUS
Orientation as to time, place, person
Personal Information ( age, birthday, address )
Fund of knowledge
President, Vice president
Knowledge of current events
Calculation
Name of objects shown ( key, pen, coin, etc. )
Response to request ( close eyes, raise hands, put R hand on L ear, etc. )
Digit span, Serial seven
Recall of 3 objects (after 1 min, 5 min, 15 min )
Recall of story
Reading
Writing ( spontaneous of 8-10 word sentence, dictation, copy )
Drawing ( shapes, clock, copy )
Gnostic functions ( recognition of objects by sight, touch, sound )
Practic functions ( show how you would comb hair, brush teeth, etc. )
Insight and judgement
Proverbs
Rote ( repetition forward and backward: counting, alphabet, days of the week )
CRANIAL NERVES
I Identifying vanilla, coffee, tobacco
II Visual acuity
Confrontation test/ Visual Field testing
Fundoscopy
II, III Pupils: Right =__ mm, Left =__mm
Direct Pupillary Light Reflex
Consensual Pupillary Light Reflex
III, IV, VI Primary gaze
Extraocular movements (EOMs)
Convergence
Accommodation
V V ( touch, pain over the 3 divisions, jaw jerk, testing temporalis and masseter tone and strength)
V, VII Corneal Reflex
VII Observe for the symmetry of movements or the presence of weakness for the ff. facial movements:
1. Wrinkle your forehead
2. Look up at the ceiling
3. Close your eyes and don’t let me open them
4. Show me your smile/show me your teeth
5. Puff out your cheeks
6. Pucker your lips
7. Pull down hard on the corners of your mouth
Taste over the anterior 2/3 of the tongue
VIII Gross hearing –rubbing hair with fingers
Weber test
Rinne test

70 | P a g e
IX, X Observing the uvula and palate
Gag reflex
Taste over the posterior 1/3 of the tongue
XI Testing the sternocleidomastoid
Shoulder shrug
XII Examination of the tongue atrophy, fasciculation, deviation, strength against cheek
Articulation
MOTOR
Shoulder Flexion, Extension, Adduction, Abduction, Internal Rotation, External Rotation
Elbow Flexion, Extension
Wrist Flexion, Extension, Pronation, Supination
Thumb Radial abduction, palmar abduction, opposition
Hip Flexion, Extension, Adduction, Abduction, Internal Rotation, External Rotation
Knee Flexion, Extension
Ankle Flexion, Extension, Inversion, Eversion
Big toe Toe (Flexion, Extension )
Walking on heels and toes
MUSCLE TONE
SENSORY
Spino- Touch
thalamic Pain
Temperature
Posterior Position sense
Column Vibratory sense
Graphesthesia
Stereognosis ( coin, key, R and L )
Two-point discrimination
Romberg’s
Double simultaneous stimulation
CEREBELLAR
Coordination (finger to nose, alternating pronation supination, check reflex, , heel to shin, tibial tapping)
Gait ( natural, tandem walk along a straight line )
Nystagmus
Hypotonia
REFLEXES
DTRs
Biceps, Triceps, Brachioradialis, Knee (Patellar), Ankle
OTHER REFLEXES ( glabellar, suck, snout, palmomental superficial abdominal, cremasteric, grasp)
Babinski
Clonus

71 | P a g e
University of the East
RAMON MAGSAYSAY MEMORIAL MEDICAL CENTER, Inc.
(UERMMMCI)
Aurora Boulevard, Quezon City

CHECKLIST OF PSYCHOMOTOR SKILLS FOR NEUROLOGY II

Name of Student: _____________________________________ Preceptor: __________________

PSYCHOMOTOR SKILLS YES NO


Able to construct a temporal profile
Able to perform a complete neurological examination (see table for neurological exam below)
Able to localize the lesion
Able to formulate an etiologic diagnosis
Able to formulate a list of differential diagnosis

NEUROLOGIC EXAMINATION YES NO


MENTAL STATUS
Orientation as to time, place, person
Personal Information ( age, birthday, address )
Fund of knowledge
President, Vice president
Knowledge of current events
Calculation
Name of objects shown ( key, pen, coin, etc. )
Response to request ( close eyes, raise hands, put R hand on L ear, etc. )
Digit span, Serial seven
Recall of 3 objects (after 1 min, 5 min, 15 min )
Recall of story
Reading
Writing ( spontaneous of 8-10 word sentence, dictation, copy )
Drawing ( shapes, clock, copy )
Gnostic functions ( recognition of objects by sight, touch, sound )
Practic functions ( show how you would comb hair, brush teeth, etc. )
Insight and judgement
Proverbs
Rote ( repetition forward and backward: counting, alphabet, days of the week )
CRANIAL NERVES
I Identifying vanilla, coffee, tobacco
II Visual acuity
Confrontation test/ Visual Field testing
Fundoscopy
II, III Pupils: Right =__ mm, Left =__mm
Direct Pupillary Light Reflex
Consensual Pupillary Light Reflex
III, IV, VI Primary gaze
Extraocular movements (EOMs)
Convergence
Accommodation
V V ( touch, pain over the 3 divisions, jaw jerk, testing temporalis and masseter tone and strength)
V, VII Corneal Reflex

72 | P a g e
VII Observe for the symmetry of movements or the presence of weakness for the ff. facial movements:
1. Wrinkle your forehead
2. Look up at the ceiling
3. Close your eyes and don’t let me open them
4. Show me your smile/show me your teeth
5. Puff out your cheeks
6. Pucker your lips
7. Pull down hard on the corners of your mouth
Taste over the anterior 2/3 of the tongue
VIII Gross hearing –rubbing hair with fingers
Weber test
Rinne test
IX, X Observing the uvula and palate
Gag reflex
Taste over the posterior 1/3 of the tongue
XI Testing the sternocleidomastoid
Shoulder shrug
XII Examination of the tongue atrophy, fasciculation, deviation, strength against cheek
Articulation
MOTOR
Shoulder Flexion, Extension, Adduction, Abduction, Internal Rotation, External Rotation
Elbow Flexion, Extension
Wrist Flexion, Extension, Pronation, Supination
Thumb Radial abduction, palmar abduction, opposition
Hip Flexion, Extension, Adduction, Abduction, Internal Rotation, External Rotation
Knee Flexion, Extension
Ankle Flexion, Extension, Inversion, Eversion
Big toe Toe (Flexion, Extension )
Walking on heels and toes
MUSCLE TONE
SENSORY
Spino- Touch
thalamic Pain
Temperature
Posterior Position sense
Column Vibratory sense
Graphesthesia
Stereognosis ( coin, key, R and L )
Two-point discrimination
Romberg’s
Double simultaneous stimulation
CEREBELLAR
Coordination (finger to nose, alternating pronation supination, check reflex, , heel to shin, tibial tapping)
Gait ( natural, tandem walk along a straight line )
Nystagmus
Hypotonia
REFLEXES
DTRs
Biceps, Triceps, Brachioradialis, Knee (Patellar), Ankle
OTHER REFLEXES ( glabellar, suck, snout, palmomental superficial abdominal, cremasteric, grasp)
Babinski
Clonus

73 | P a g e
APPENDIX
MINI-MENTAL STATUS EXAMINATION – screening tool for memory loss

74 | P a g e
75 | P a g e
Montreal Cognitive Assessment (MoCA) - screening tool for memory loss

76 | P a g e
77 | P a g e
78 | P a g e
79 | P a g e
80 | P a g e
MOCA (FILIPINO VERSION)

81 | P a g e
82 | P a g e
83 | P a g e
84 | P a g e
85 | P a g e
86 | P a g e
Glasgow Coma Score

Behavior Response Score


Eye Opening Response Spontaneous 4
To Speech 3
To Pain 2
No Response 1
Verbal Response Oriented 5
Confused 4
Inappropriate words 3
Incomprehensible sounds 2
No response 1
Motor Response Obeys commands 6
Moves to localized pain 5
Flexion withdrawal from pain 4
Abnormal flexion (decorticate) 3
Abnormal extension (decerebrate) 2
No response 1

References
Teasdale G, Jennett B. Assessment of coma and impaired consciousness. Lancet 1974; 81-84.
Teasdale G, Jennett B. Assessment and prognosis of coma after head injury. Acta Neurochir 1976; 34:45-
55.

RECOMMENDED TEXTBOOKS and REFERENCE MATERIALS

www.neuroexam.com/neuroexam
Adam’s Principles of Neurology
DeMyer
Clinical Neurology through Clinical Cases

87 | P a g e
CLINICAL CASES FOR LOCALIZATION
Neurology I

CASE 1

A 70-year-old man woke up one morning and could not move his right arm or hand. He had trouble
trying to speak to his wife, but could understand her questions and follow her directions. At the hospital,
neurological exam revealed the following abnormalities:

• paralysis of right lower face muscles (but he could wrinkle his forehead on both sides)
• 3/5 motor paralysis and hyperreflexia of right hand and arm
• mild sensory loss (both modalities) in right hand, arm and face
• decreased verbal output, intact comprehension, impaired naming and impaired repetition
• Preferential gaze to the left

Question 1: What structures are involved?


Question 2: What is the most likely location for the lesion?
Question 3: Does this patient have aphasia? If so, what kind?
Question 4 Differentiate the kinds of aphasia

88 | P a g e
CASE 2

A 65-year-old womman with a history of poorly controlled hypertension woke up and fell on the floor
as she tried to stand up from the bed. She was immediately brought to the emergency room where
neurological examination revealed the following:

• 2/5 paralysis of left arm, leg and lower face


• Upgoing toe on the left
• complete sensory loss in left arm, leg and face

Question 1: What structures are involved?


Question 2: What is the most likely location for the lesion?
Question 3: If the patient had a lesion in the parietal lobe, what kind of visual field defect would you
expect?

89 | P a g e
CASE 3

A 39-year-old woman went to see her family physician complaining of weakness in her legs. She
said her legs felt tired when she stood up too long. She first noticed the weakness three months earlier, a
few weeks after she slipped in the bathroom and fell on her buttocks. Since then, there was noted
intermittent, shock-like pain when she drove through traffic or when she sat too long. Examination
revealed increased reflexes in both legs and bilateral Babinski signs. There was marked weakness of foot,
ankle and calf muscles, but thigh muscles did not show definitive weakness. Sensory examination revealed
mild deficits in all modalities in both lower legs.

Question 1: Which of the major long tracts are involved?


Question 2: What is the most likely location for the lesion?

90 | P a g e
CASE 4

A 65-year-old man presented with loss of all sensation in the left thumb, adjacent hand and strip
along the lateral forearm. There were no other abnormal findings on neurological exam.

Question 1: Where is the lesion?


Question 2: Draw the distribution of the lesion.

91 | P a g e
CASE 5

A 57-year-old man complained of sudden onset doubling of vision after a traffic altercation. At
the emergency room, the blood pressure was 220/100mmHg. Neurological examination revealed inability
to close his left eye, inability to wrinkle the left side of his forehead, and no movement on the left side of
his face when he tried to smile. He also noted decreased taste sensation on the left side of the tongue.
His right showed unremarkable movement. His left eye was noted to be deviated towards the nose and
he was unable to abduct the left eye. His right upper and motor extremities had motor strength of 4/5.

Question 1: What structures are involved?


Question 2: What is the most likely location for the lesion?
Question 3: Can the findings be explained by one lesion? Why or why not?

CASE 6

92 | P a g e
A 47-year-old man complained of facial asymmetry on the right for the past two days. History
started 5 days ago when he had sore throat, nasal congestion and low-grade fever. Three days ago, he
complained of 5/10 pain over the right temporal area relieved by Paracetamol. Two days ago, he woke up
and his daughter noted that his right eyelid was not closing and he drooled on the right side of his mouth
when he drank water. Neurological examination revealed inability to close his left eye, inability to wrinkle
the left side of his forehead, and no movement on the left side of his face when he tried to smile. Taste
was also impaired on the left side of the tongue. Examination of hearing was normal. The rest of the
neurological examination was unremarkable.

Question 1: Is this a central or peripheral nervous system problem?


Question 2: Draw the structure involved and identify the exact location of the lesion.

CASE 7

93 | P a g e
A 42-year-old woman with known history of rheumatic heart disease and paroxysmal atrial
fibrillation had sudden onset, severe dizziness described as a sensation of spinning which did not resolve
upon closing the eyes and episodes of vomiting. Upon examination, there was difficulty evaluating the
eyes as she preferred them closed but was noted to have bilateral nystagmus on primary gaze. Motor
strength and sensory examination were unremarkable although there was slight hypotonia on the left
upper and lower extremities. There was dysmetria and dysdiadochokinesia on the left. Speech was
dysarthric and she was talking “as if she was drunk”.

Question 1: What structures are involved?


Question 2: What is the most likely location for the lesion?
Question 3: Can the findings be explained by one lesion? Why or why not?

CASE 8

94 | P a g e
Refer to the patient in Case 7. On further neurological examination, the patient had additional
findings of: loss of pain and temperature sensation on the left side of her face and left side of her body,
hoarseness, and difficulty swallowing liquids.

Question 1: What structures are involved?


Question 2: What is the most likely location for the lesion?
Question 3: If this were a vascular lesion, which blood vessels would be involved?

CASE 9

95 | P a g e
A 55-year-old man complained of weakness of the right arm and leg and diplopia. . Examination
revealed hyperreflexia and Babinski sign on the right, and weakness of the right lower face. The left eye
showed ptosis, dilated pupil, and inability to adduct.

Question 1: What structures are involved?


Question 2: What is the most likely location for the lesion?
Question 3: If this were a vascular lesion, which blood vessels would be involved?

CASE 10

96 | P a g e
A 54 year old man was referred for progressive weakness over the past 8 months. Weakness
began as a foot drop in the left lower extremity, and similar symptoms developed 2 months later in the
right leg. The mental status was unremarkable. There were noted fasciculations on the right and left sides
of the tongue. There is slight atrophy in the intrinsic hand muscles bilaterally, but motor strength was 5/5.
In the lower extremities, there was spasticity with prominent wasting and fasciculations in all muscles
below the knees. There is mild weakness at 4+/5 in hip flexion, extension, abduction and adduction. Exam
of the more distal muscles shows marked bilateral foot drop as well as weakness of plantar flexion, ankle
inversion and eversion. The reflexes were normal in the upper extremities but the lower extremities
showed clonus and bilateral Babinski. Sensory exam: normal to light touch, temperature and vibration.
Coordination: No tremor, or dysmetria in the upper extermities. No ataxia. Gait: slow, spastic, and
slapping.

Question 1: What structures are involved?


Question 2: What is the most likely location for the lesion?

CASE 11

97 | P a g e
Differentiate the types of dysconjugate eye movements (walleyed syndrome; vertical one and a
half syndrome; internuclear ophthalmoplegia; parinaud syndrome; one and a half syndrome) by:

1. Describing the abnormality during extraocular movement


2. Drawing the abnormality, the direction of the eyes at primary gaze
3. Identifying the structures involved in the abnormality
4. Enumerating the most common neurological conditions that would cause these findings

CASE 12

98 | P a g e
Differentiate the types of pupillary abnormalities: Horner Syndrome; Argyll-Robertson Pupil; Adie
Tonic Pupil; Marcus Gunn Pupil) by:

1. Describing the abnormality during testing


2. Drawing the abnormality
3. Identifying the structures involved in the abnormality
4. Enumerating the most common neurological conditions that would cause these findings

CLINICAL CASES
99 | P a g e
Neurology II

CASE 1

A 70-year-old man woke up one morning and could not move his right arm or hand. He had trouble
trying to speak to his wife, and could not understand her questions and follow her directions. At the
hospital, neurological exam revealed the following abnormalities:

• paralysis of right lower face muscles (but he could wrinkle his forehead on both sides)
• 3/5 motor paralysis and hyperreflexia of right hand and arm
• mild sensory loss (both modalities) in right hand, arm and face
• absent verbal output, absent comprehension, absent naming and absent repetition
• Preferential gaze to the left
• Absent visual threat on the right

Question 1: What structures are involved?


Question 2: What is the most likely location for the lesion?
Question 3: Does this patient have aphasia? If so, what kind?
Question 4: Differentiate the kinds of aphasia?
Question 5: What is the most likely etiology of the lesion? Give differential diagnoses for this case.
Question 6: What diagnostic procedures would you request for this patient?

CASE 2

100 | P a g e
A 55-year-old man, ship captain was in good health until 8 months ago when he noted the
development of a tremor involving the right hand. He noted the tremors to be more prominent when he
was stressed and would disappear spontaneously. Sometimes, he would notice the tremor when his hand
was on the table while he was doing paperwork. He consulted a neurologist because he was disturbed by
these symptoms. He has no other complaints. On examination, there is a tremor in the right arm at rest
and while he walks. He has a sustained tremor in both arms, and a fine tremor during finger-nose
examination. His gait is fairly unremarkable. Tone is increased in the right arm and leg. The physical
examination is otherwise unremarkable. He and his wife deny his use of alcohol or any other medications.

Question 1: What structures are involved?


Question 2: What is the most likely location for the lesion?
Question 3: What is the most likely etiology of the lesion? Give differential diagnoses for this case.
Question 4: What diagnostic procedures would you request for this patient?
Question 5: What is the next step in therapy?

CASE 3

101 | P a g e
A 22-year-old medical student was studying late at night for an examination, talking to his friends.
All of a sudden, his friends noted that he began smacking his lips, stared into space, seemed confused,
and kept mumbling the same word repeatedly. During this time, his fingers were moving “as if they were
fidgeting”. This episode lasted approximately 30 seconds. During the episode, his friends tried to talk to
him, but he was unresponsive. Within a few more seconds, he suddenly became asymptomatic although
he seemed slightly confused for 5 to 10 more seconds. He was brought to the emergency room where the
symptoms recurred. However, instead of resolving, he was noted to have progression towards generalized
tonic-clonic jerking of the upper and lower extremities which persisted for the next 5 minutes.

Question 1: How would you manage the patient emergently?


Question 2: Once the patient is stabilized, how would you proceed with diagnostic management?
Question 3: What is the next step in therapy?

CASE 4

102 | P a g e
A 27-year-old female consults due to headaches of increasing frequency. Upon interview, she has
a 10-year history of headaches which started in College. She would experience headaches anywhere from
2 – 4 times a month especially during the time of her menstruation and when she lacked sleep. The
headache starts behind the right eye, throbbing in character and is usually preceded by flashing lights and
zigzag lines. Once the headache occurs, it progresses in intensity. There is extreme nausea and vomiting,
and the patient goes into a dark room to minimize her head pain. She claims that vomiting relieves her
and she needs to “sleep it off” to relieve her headaches as she did not like taking pain relievers. Generally,
the headache lasts 4 to 6 hours, but the patient feels tired and listless for the next 24 hours. The patient
feels that the headache worsens with her menstrual cycle, and certain foods especially red wine can
exacerbate her headache. Six months ago, she started taking combination low-dose oral contraceptive
pills. Since then, she noted increased frequency of headache to almost daily episodes. She discontinued
the pills but the headaches remain frequent.
Her general and neurologic examinations are normal.

Question 1: What structures are involved?


Question 2: What is the most likely location for the lesion?
Question 3: What is the most likely etiology of the lesion? Give differential diagnoses for this case.
Question 4: What diagnostic procedures would you request for this patient?
Question 5: What is the next step in therapy?

CASE 5

103 | P a g e
A 28-year-old male, construction worker was brought to the emergency room due to 1 week
complaint of headache. The headache is primarily in the frontal and occipital regions and associated with
mild nausea and lightheadedness. The mother also noted low-grade, undocumented fever and dry cough.
He has taken various paracetamol 500mg every 4 hours without any improvement in the headache. The
intensity of the headache has gradually increased since it began prompting evaluation as he was no longer
able to tolerate it. He also noted neck pain and stiffness about 3 days ago.
On examination, he has a temperature of 38.4°C; blood pressure, 110/70 mmHg; and pulse, of
102 beats/min. He is awake and alert and fully oriented. His Mental Status Examination is normal. Cranial
nerves are normal except for bilateral papilledema. The deep tendon reflexes are 3+ throughout without
evidence of a Babinski sign. His motor, sensory, and cerebellar examinations are normal. Brudzinski and
Kernig signs were present.

Question 1: What structures are involved?


Question 2: What is the most likely location for the lesion?
Question 3: What is the most likely etiology of the lesion? Give differential diagnoses for this case.
Question 4: What diagnostic procedures would you request for this patient?
Question 5: What is the next step in therapy?

CASE 6

104 | P a g e
A 36-year-old woman consulted at the Neurology OPD clinic complaining of severe headache of 1
month duration. She has been experiencing headaches almost 1 month prior to consult. Her headaches
were described as generalized and dull, involving the entire head. Initially, the pain scale was 3-4/10 but
over the past 2 weeks, she noted increasing severity of the headache, now at 8-10/10. She has not been
able to report to work as a saleslady. She has tried taking paracetamol, then celecoxib for the headache
but these afforded very little relief. For the past week, she noticed transient blurring of vision described
as “graying-out” of vision when she defecates or when she lifts pails of water. On review of history, she
started taking oral contraceptive pills for birth control about 6 months prior to the onset of symptoms
On examination, vital signs were unremarkable. There are no cranial bruits, and her cardiovascular
examination is normal. Her neurologic examination is notable for bilateral papilledema with intact visual
acuity and intact extra-ocular muscles. Her visual fields are normal. She has normal color perception. The
rest of the neurological examination was unremarkable.

Question 1: What structures are involved?


Question 2: What is the most likely location for the lesion?
Question 3: What is the most likely etiology of the lesion? Give differential diagnoses for this case.
Question 4: What diagnostic procedures would you request for this patient?
Question 5: What is the next step in therapy?

CASE 7

105 | P a g e
A 28-year-old male, known HIV-positive and currently on antiretroviral treatment, consults at the
emergency room with right facial paralysis of 1 day duration. Three days ago, he began to experience right
ear pain, throbbing and tender to the touch, around 4/10 in pain score. He did not take any pain
medication and has not had any fever. Yesterday, he was able to feel and see small, reddish blisters on
his right earlobe. This morning, he woke up and noted that the right side of his face was not moving and
that sounds seemed to be muffled on his right ear. Water and saliva drooled over the right side of his
mouth and he noted impairment of taste over the right side of his tongue as he was eating breakfast.
He does have a past history of chicken pox as a child.
His physical examination shows small blisters on an erythematous base in the right external ear.
The examination of the ear canal is painful to her, but the tympanic membrane is intact. No pus is seen in
the ear canal. The left ear canal is normal.
His neurological examination shows right peripheral facial palsy, decreased taste on the right and
decreased hearing on gross testing over the right ear. The Weber tuning fork test lateralizes to the left
ear. The Rinne test is normal in both ears. Taste is impaired on the right side of the tongue. The rest of
the neurological examination is normal.

Question 1: What structures are involved?


Question 2: What is the most likely location for the lesion? Draw the pathway of the cranial
nerve/nerves involved.
Question 3: What is the most likely etiology of the lesion? Give differential diagnoses for this case.
Question 4: What diagnostic procedures would you request for this patient?
Question 5: What is the next step in therapy?

CASE 8

106 | P a g e
A 65-year-old male consulted at the Neurology OPD clinic due to progressive weakness of 10
months. He first noticed weakness of his right hand with difficulty and clumsiness holding on to mugs and
kitchen utensils. After about 2 months, the symptoms progressed and the patient began to experience
difficulty raising his arm above his head or carrying things. In the past 4 months, he noted that, sometimes,
he would choke when drinking water but still had good appetite.
On neurological examination, mental status was intact. Examination of the tongue showed
atrophy of the right side of the tongue as well as fasciculations on the surface of the tongue. There were
findings of muscle atrophy and wasting of the intrinsic and small muscles of his right hand, right triceps,
and muscles of his right shoulder. There was visible muscle twitching of both arm muscles and paraspinal
muscles of his back. The neurologic examination showed 3/5 motor strength of the right upper extremity
and 4/5 motor strength on his left proximal upper extremity. Reflexes were 3+ on all extremities. His
sensory and cerebellar examinations were normal. MRI of the brain and spine were normal. Laboratory
studies were normal. Electrodiagnostic studies (EMG/NCV) reveal diffuse muscle denervation in his arms,
legs, and paraspinal muscles. There was no evidence of neuropathy or myopathy.

Question 1: What structures are involved?


Question 2: What is the most likely location for the lesion?
Question 3: What is the most likely etiology of the lesion? Give differential diagnoses for this case.
Question 4: Are there other diagnostic tests that you would like to request?
Question 5: What is the next step in therapy?

CASE 9

107 | P a g e
A 48-year-old female, left-handed office secretary consults due to numbness and pain of both
hands. 1 year prior to consult, she began to experience intermittent numbness and pain of her left index
and middle fingers, described as pins-and-needles sensation as well as a feeling of cold. These were
experienced typing on her keyboard and when she performed household chores like chopping, doing the
laundry and cleaning the dishes. These were experienced intermittently until 6 months ago when she
started experiencing the same symptoms on her right hand also. The numbness and pain involved her left
index and middle fingers. At about this time, she noted that the pain and numbness over the left hand
became increasingly frequent and often awakened the patient from sleep. She has also recently noticed
decreased grip strength associated with frequent dropping of heavy objects when she used her left hand.
Her neurologic and physical examinations are significant for numbness to pinprick sensation along
the lateral sides of both her palm, thumb, index and middle fingers. The findings were more pronounced
on the left. Tinel and Phalen’s sign were present bilaterally, with the left worse than the right. The rest of
her examination, including muscle and sensory testing are normal. Her deep tendon reflexes are normal,
throughout. There are no musculoskeletal or joint abnormalities observed.

Question 1: What structures are involved?


Question 2: What is the most likely location for the lesion? Draw the pathway of the nerve/nerves
involved.
Question 3: What is the most likely etiology of the lesion? Give differential diagnoses for this case.
Question 4: What diagnostic procedures would you request for this patient?
Question 5: What is the next step in therapy?

CASE 10

108 | P a g e
A 28-year-old woman presents at your clinic with a 3-month history of muscle soreness, cramps,
and muscle fatigue with climbing stairs and carrying objects. She initially thought that this was due to
fatigue and sleep deprivation due to her work at an ad agency. Two weeks ago, she went on vacation but
the symptoms persisted. A week ago, she noted reddish rashes on her cheeks, necks, chest, and back and
swelling around her eyes.
Physical examination shows erythematous rash across her cheeks, neck, chest, and back and mild
lid edema. The cardiac exam is significant for occasional skipped beats but otherwise unremarkable. The
neurologic examination shows proximal muscle weakness of the patient’s deltoids, biceps, hip flexors, and
knee flexors at 4/5. Mental status, cranial nerves, sensory and cerebellar examination is normal.
Laboratory studies are normal except for elevated serum creatine kinase of 1770 (normal 50–200).

Question 1: What structures are involved?


Question 2: What is the most likely location for the lesion?
Question 3: What is the most likely etiology of the lesion? Give differential diagnoses for this case.
Question 4: What diagnostic procedures would you request for this patient?
Question 5: What is the next step in therapy?

109 | P a g e

You might also like